SlideShare a Scribd company logo
1 of 71
Download to read offline
ABDULRAHMAN BASHIRE
BEGHAZI CHILDREN HOSPITAL
11/12/2019
PEDIATRIC IMMUNIOLOGY MCQ
2 ABDULRAHMAN BASHIR BENGHAZI CHILDREN HOSPITAL
2
25.1.2018
‫اھﺪاء‬
‫ان‬ ‫ﷲ‬ ‫ﻣﻦ‬ ‫راﺟﯿﺎ‬ ‫اﻻطﻔﺎل‬ ‫اطﺒﺎء‬ ‫ﺟﻤﯿﻊ‬ ‫اﻟﻰ‬ ‫اﻟﻌﻤﻞ‬ ‫ھﺬا‬ ‫اﻗﺪم‬
‫ﯾﻨﺎل‬‫ﺑﺎﻟﺮﺣﻤﺔ‬ ‫اﻟﺪﻋﺎء‬ ‫اﻻ‬ ‫وﻻاطﻠﺐ‬ ، ‫ورﺿﺎﺋﮭﻢ‬ ‫اﻋﺠﺎﺑﮭﻢ‬
‫واﻟﻤﻐﻔﺮة‬‫روﺣﻰ‬ ‫وﺗﺆاﺋﻢ‬ ‫وﻋﻀﺪى‬ ‫ﻻﺧﻮى‬
"‫و‬ ‫اﺣﻤﺪ‬‫ﻋﺒﺪاﻟﻤﻨﻌﻢ‬"
‫ﻟ‬ ‫اﻏﻔﺮ‬ ‫اﻟﻠﮭﻢ‬‫ﮭﻢ‬‫ﺟﻨﺎﻧﻚ‬ ‫ﻓﺴﯿﺢ‬ ‫واﺳﻜﻨﮭﻢ‬ ‫وارﺣﻤﮭﻢ‬.
‫اﻣﯿﻦ‬
2 ABDULRAHMAN BASHIR BENGHAZI CHILDREN HOSPITAL
3
25.1.2018
Regarding HiB vaccination, which of the following are true?
A. It is a live vaccine.
B. It is contra-indicated in a child with a history of convulsions
C. A booster is given with the MMR immunisation within one month of the first birthday.
D. Vaccinations are given at 2, 3 and 4 months plus booster between 12 & 13 months.
E. It must not be given as a single injection with DPT
Ans:-CD
DTaP/IPV/Hib is a combined vaccine which contains the following active ingredients: purified high dose
diphtheria toxoid; purified tetanus toxoid; five purified components of the Bordetella pertussis bacteria;
three strains of inactivated polio virus and a purified component of Haemophilus infiuenzae type b bacteria
attached to a tetanus toxoid carrier protein. It is usually given to babies at the age of two, three and four
months and again as a booster between 12 and 13 months.
Since introduction of the HiB vaccine, the incidence of which of the following have conditions have
decreased?
A. Meningitis
B. Otitis media
C. Septic arthritis
D. Epiglottitis
E. Periorbital cellulitis
Ans:-ACDE
The HiB vaccine primarily prevents invasive HiB disease. In the past it had been a major cause of
meningitis, septic arthritis, epiglottitis and peri-orbital cellulitis. It is logical therefore that the incidence of
these diseases has declined since the introduction of the vaccine. Haemophilus influenzae type B is not
recognised as a major cause of otitis media thus it is unlikely that the overall incidence of otitis media has
been affected by the vaccine.
The following are live vaccines
A. BCG.
B. Measles.
C. Oral polio
D. Pertussis.
E. Rubella
Ans:-ABCE
Which vaccination is given by the intradermal route usually?
A. Inactivated Polio
B. Meningococcal C
C. MMR.
D. DTP.
E. BCG
Ans:-E
BCG is the only vaccine given intradermally. The rest are given by the intramuscular or deep subcutaneous
route
Which of the following are true regarding vaccines?
A. Diptheria vaccine is a toxoid.
2 ABDULRAHMAN BASHIR BENGHAZI CHILDREN HOSPITAL
4
25.1.2018
B. Pertussis vaccine is a killed bacillus.
C. Injected polio vaccine is a live attenuated virus.
D. BCG is a live attenuated bacillus.
E. Measles is a live attenuated virus
Ans:-ADE
Previously virulent micro-organisms that have been inactivated (with chemicals or heat) include vaccines
against influenza, cholera, plague, and hepatitis A. Most such vaccines may have incomplete or short-lived
immune responses and are likely to require booster shots.
Live, attenuated vaccines typically provoke more durable immunological responses. Examples include
yellow fever, measles, rubella, and mumps. BCG is a live tuberculosis strain. The oral polio vaccine is a
live attenuated virus but injected polio vaccine is inactivated.
Inactivated toxic compounds (Toxoids) from micro-organisms include tetanus and diphtheria. The pertussis
vaccine is acellular and made from part of the pertussis cell.
Regarding immunisation in the UK:-
A. The first dose of vaccines is given at a corrected age of 2 months in case of premature delivery
B. Family history of febrile convulsion following immunisation is a relative contraindication
C. Measles, mumps, rubella and polio are contraindicated in HIV +ve children
D. Live vaccines should be avoided within 6 months of termination of lymphoma treatment
E. Prolonged inconsolable crying to previous immunisation is a definite contraindication
Ans:- DE
Prematurity, asthma, chronic lung disease, congenital heart disease, and Downs syndrome are associated
with an increased risk of complications from infectious diseases and they should be immunised as a matter
of priority. Premature babies should be immunised according to the recommended schedule from 2 months
after birth, irrespective of prematurely. Studies have shown that antibody response and adverse events are
not significantly different in preterm and term infants immunised 2,3, and 4 months afterbirth. Prolonged
inconsolable crying or high pitched screaming more than 4 hours is classified as a severe reaction to
immunisation and is a contraindication to subsequent immunisation with same vaccine. Patients on active
chemo or radiotherapy or within 6 months of termination of such treatment are contraindicated for live
vaccines. They may not be able to produce a normal immune response to live vaccines and could
potentially suffer from severe manifestation such as disseminated infection with BCG or paralytic polio
from vaccine virus. No harmful effects have been reported with MMR and polio in HIV +ve individuals.
However it should be noted that Polio is secreted for longer periods in HIV positive patients as compared to
other people. Family history of febrile convulsion is not a contraindication to immunization
A live vaccine shouldn’t be given to patient who :-
A. Is finishing course of antibiotic therapy
B. Received pooled immunoglobulin 2 month ago
C. Has previous history of infection
D. Is receiving high dose of corticosteroid
E. Finished treatment for Hodgkin's disease 5 years ago
Ans:-BD
Live vaccine shouldn’t be given in the following circumstances:-
Patient who have received chemotherapy or radiation within last year.
Patient with malignant disorder such as Leukemia or lymphoma .
Patient with immunosuppressive disorder
Those receiving high dose of corticosteroid or immunoglobulin within the last 3 month
It is appropriate to give live vaccine in the following;
 Non-febrile common cold
 Antibiotic treatment
2 ABDULRAHMAN BASHIR BENGHAZI CHILDREN HOSPITAL
5
25.1.2018
 Previous history of the infection
 Stable neurological condition
If the patient had an acute illness the vaccination should be postponed until the fever has resolved
The following are true regarding vaccine administration in children:-
A. Live vaccines are contraindicted within 3 weeks of another live vaccine administration
B. MMR can be given a week after administration of IV immunoglobulins
C. Live vaccines are not given to children on prolonged steroid therapy
D. Severe febrile illness is a contraindication for DPT
E. Severe malnutrition is a contraindication for BCG
Ans:-ACD
Live vaccines are contraindicated 3 weeks before and 6 weeks after administration of Immunoglobulins.
Other contraindications to live vaccines include –steroid administration / immunosuppressive therapy/
severe egg allergy/ immunodeficient conditions.
Contraindications to pertussis vaccination include hypersensitivity to the vaccine and a severe reaction such
as shock, persistent screaming, fever over 40.5 degree or serious neurological symptoms.
Malnutrition, Stable neurological conditions, minor illness, chronic diseases of heart, lung and kidney,
antibiotic treatment are not contraindications to vaccines.
Which vaccination is NOT included in the 2, 3, 4 month ages of a child according to the UK
immunisation schedule?
A. Polio.
B. Hib.
C. DTP.
D. MeningococcalC
E. BCG
Ans:-E
BCG is usually given at birth in high risk neonates or at ages 10-14. Universal BCG immunisation of
teenagers has recently been removed from the immunisation schedule, with a targeted strategy now
recommended to immunise groups at increased risk in infancy, at birth or adulthood.
Which of the following are true regarding BCG vaccination?
A. Efficacy is demonstrated by a hypersensitivity response to vaccine administration.
B. It is contraindicated in neonates.
C. It offers protection against infection with Mycobacterium bovis as well as Mycobacterium TB.
D. It should be injected subcutaneously.
E. It contains a live attenuated virus
Ans:-C
Bacillus of Calmette and Guerin (BCG) is a vaccine against tuberculosis that is prepared from a strain of
the attenuated (weakened) live bovine tuberculosis bacillus, Mycobacterium bovis (a bacterium, not a
virus), that has lost its virulence in humans through serial propagation in artificial media. The bacilli retain
sufficient immunogenicity to behave as a reasonably effective vaccine for the prevention of human
tuberculosis, particularly in the prevention of TB meningitis and disseminated disease in early childhood.
Protection against pulmonary disease in later life is less certain. BCG vaccine should always be given
intradermally as subcutaneous administration is likely to be associated with adverse effects including local
abscess formation and suppurative lymphadenitis. The Mantoux test results in a cutaneous hypersensitivity
response to administered PPD (purified protein dericative), but neither a cutaneous response to the vaccine
or to PPD are sufficient or required to demonstrate vaccine efficacy.
With regard to Bacillus Calmette-Guerin (BCG):
A. High-risk infants should have a positive Heaf test before immunisation proceeds.
2 ABDULRAHMAN BASHIR BENGHAZI CHILDREN HOSPITAL
6
25.1.2018
B. It is safe in asymptomatic HIV-positive patients.
C. It may be given at the same time as other live vaccines.
D. It should be given after a positive tuberculin test.
E. Offers some protection against leprosy
Ans:-CE
High-risk infants do not require skin testing before immunisation up to the age of 3 months. BCG should be
given only after a negative tuberculin test. It can be given at the same time as other live vaccines, otherwise
a gap of 3 weeks should be observed. There is a risk that a suboptimal response to both may occur if this
gap is not observed or the injections are not given at the same time. Live oral polio vaccine (OPV), which
works by inducing gut immunity, is the exception and can be given at any time. HIV infection is an
absolute contraindication to BCG vaccination.
Which of the following children should be offered BCG immunisation? (Assume that any skin tests,
where appropriate, have been done and proved negative)
A. Normal healthy infants with Asian mothers going to Bangladesh.
B. Migrant from Africa.
C. A 13-year-old child who is moving to a high risk area but who has had immunisation as an infant.
D. Child who has a contact with active pulmonary TB.
E. Infant with a grandparent born in a high risk area.
Ans:-ABDE
BCG is recommended for the following groups if BCG immunisation, as evidenced by a characterstic scar,
has not previously been carried out and they are negative for tuberculoprotein hypersensitivity:
• all infants living in areas where the incidence of tuberculosis is greater than 40 per 100 000;
• infants with a parent or grandparent born in a country with an incidence of tuberculosis greater than 40
per 100 000;
• previously unvaccinated new immigrants from countries with a high incidence of tuberculosis;
• contacts of those with active respiratory tuberculosis;
• health service staff
• veterinary staff
• staff working in prisons, in residential homes and in hostels for refugees and the homeless;
• those intending to stay for more than 1 month in countries with a high incidence of tuberculosis
• neonates, infants, children or adults where immunisation is requested.
A tuberculin skin test is necessary prior to BCG vaccination for:
• all individuals aged six years or over
• infants and children under six years of age with a history of residence or prolonged stay (more than three
months) in a country with TB.
• those who have had close contact with a person with known TB.
• those who have a family history of TB within the last five years.
Regarding tuberculin testing:
A. It traditionally involves an injection into the extensor surface of the left forearm.
B. The Heaf test should ideally be read between 48 and 72 hours.
C. A positive result occurs when the area of induration is > 5 mm.
D. Is negative if the Heaf grade is 1.
E. Induration > 15 mm requires further investigation and possible antituberculous chemotherapy
Ans:-CDE
Tuberculin testing traditionally involves an injection into the flexor surface of the left forearm. The Heaf
test is ideally read at 7 days (between 3 and 10 days) and the Mantoux test is read at 48-72 hours (but up to
96 hours). A positive result occurs when the area of induration is > 5 mm. N.B. the area of' flare' is
irrelevant.
2 ABDULRAHMAN BASHIR BENGHAZI CHILDREN HOSPITAL
7
25.1.2018
The Heaf test is graded 0-4. Heaf grades 0-1 are negative and grades 2-4 are positive. Strongly positive
reactions (i.e. Heaf grade 3-4 or induration > 15 mm) require further investigation and possible
antituberculous chemotherapy.
---------------------------------------------------------------------------------------------------------------------------------
MMR contraindications include which of the following?
A. Child treated with 2mg/kg/day prednisolone for at least one week in the past month.
B. Children with history of neonatal seizures.
C. Previous vomiting after egg ingestion.
D. History of febrile convulsions.
E. Severe eczema
Ans:-A
Contraindications include: acute fever, recent immunoglobulin administration, administration of another
live vaccine in the past 3 weeks, immunosuppression, allergy to neomycin or kanamycin, pregnancy.
Steroids, such as prednisolone, given to children either orally or rectally at a daily dose of 2 mg/kg/day for
at least one week or 1 mg/kg/day for one month should be deemed as a contraindication and administration
on live vaccines should be postponed for at least 3 months after immunosuppressive treatment has stopped.
In children with a significant history of an anaphylactic reaction to eggs, or who have had egg allergy and
chronic severe asthma the vaccination should be administered in hospital. The vaccine is not however
contraindicated in such patients.
Which of the following is NOT a contra-indication to MMR vaccination:
A. Children receiving high-dose cortico-steroids.
B. Children with an anaphylactic allergy to egg.
C. Children with an allergy to neomycin.
D. Children with an acute febrile illness.
E. Children who have received an immunoglobulin injection within the last 3 months
Ans:-B
All of the above are contra-indications as written in the BNF except egg allergy - this is a common
misconception. In children with a significant history of an anaphylactic reaction to eggs, or who have had
egg allergy and chronic severe asthma, the vaccination should be administered in hospital. The vaccine is
not however contraindicated in such patients.
Contraindications include: acute fever, recent immunoglobulin administration, administration of another
live vaccine in the past 3 weeks, immunosuppression, allergy to neomycin or kanamycin, pregnancy.
Steroids, such as prednisolone, given to children either orally or rectally at a daily dose of 2mg/kg/day for
at least one week or 1 mg/kg/day for one month should be deemed as a contraindication and administration
on live vaccines should be postponed for at least 3 months after immunosuppressive treatment has stopped.
Which statements concerning the MMR vaccination are correct?
A. Previous measles is a contraindication.
B. The first dose is given at 6 months.
C. A booster dose is given at 3-5 years.
D. Parotid swelling is a recognised side effect
E. Idiopathic thrombocytopenic purpura is a rare complication
Ans:-CDE
Any child who has had measles, mumps or rubella should be given the MMR vaccination regardless of
previous infection (BNF). The first dose is given at 12-15 months with a pre-school booster. ITP is a rare
complication and the risk of developing ITP is much less with MMR than with the actual diseases of
measles, mumps and rubella. Parotid swelling is also a recognised side effect.
With regard to the Measles, Mumps and Rubella (MMR) vaccine:
2 ABDULRAHMAN BASHIR BENGHAZI CHILDREN HOSPITAL
8
25.1.2018
A. It should not be given if there is a history of previous measles infection.
B. It should not be given if there has been a previous measles immunization.
C. It may be contraindicated in a child on oral corticosteroids.
D. It is contraindicated where there is a personal history of febrile convulsions.
E. It is contraindicated if there is a family history of epilepsy
Ans:-C
Live vaccine is contraindicated in children who may be immunosuppressed (such as oral steroids). Steroids,
such as prednisolone, given to children either orally or rectally at a daily dose of 2mg/kg/day for at least
one week or 1mg/kg/day for one month should be deemed as a contraindication and administration of live
vaccines should be postponed for at least 3 months after immunosuppressive treatment has stopped.
Previous infection or immunisation is not a contraindication, and 2 MMR doses are recommended. A fit
within 72 hours of a previous dose would be a contraindication, although febrile convulsions and a family
history of epilepsy are not.
The measles-mumps-rubella (MMR) vaccine is:
A. Contraindicated in children with egg allergy.
B. A live vaccine.
C. Contraindicated in children with autistic spectrum disorder.
D. Recognised as having more side effects after the second dose.
E. Contraindicated in a child with Di George syndrome
Ans:-BE
The MMR vaccine is a live vaccine and therefore is contraindicated in children with impaired cell-mediated
immunity, as in Di George syndrome. Other contraindications for live vaccines include prednisolone (orally
or rectally) at a daily dose of 2 mg/kg per day for at least 1 week or 1 mg/kg per day for a month, lower
doses of steroid if used in conjunction with cytotoxic drugs, immunosuppression secondary to an
underlying disease, immunoglobulin administration in the previous 3 months, children who have had a
bone marrow transplant within 6 months and children being treated for malignant disease with
chemotherapy /radiotherapy or who have completed treatment in the past 6 months. MMR is not
contraindicated in egg allergy or in children with autistic spectrum disorder. Side effects include malaise,
fever, rash within 7-10 days, parotid swelling 1%, febrile convulsion 1/1000. Arthropathy/
thrombocytopenia are rare. All side effects are less common after the second dose. Current immunisation
information can be found.
The MMR vaccine:
A. Is contraindicated in patients allergic to neomycin.
B. Should not be given within 3 weeks of another live vaccine (except oral polio vaccine).
C. Is safe in pregnancy.
D. Commonly results in a rash with or without fever from day 5 to day 10, lasting approximately 2 days.
E. Is contraindicated in patients who have received an injection of immunoglobulin within 3 months
Ans:-ABDE
The MMR vaccine is contraindicated in patients who are allergic to neomycin or kanamycin. In children
who have had a previous anaphylactic reaction to egg, immunisation is not absolutely contraindicated and
should be discussed with a local paediatrician or immunisation coordinator. The MMR vaccine should not
be given within 3 weeks of another live vaccine, because this results in a suboptimal response. Likewise, it
is contraindicated in patients who have received an injection of immunoglobulin within 3 months, because
no response will be mounted in the presence of immunoglobulin that may contain antibodies to measles,
mumps or rubella. Pregnancy should be avoided for at least 1 month after immunisation, which may well
result in a rash with or without fever from about day 5-10 lasting about 2 days. It is therefore sensible to
provide advice on temperature control at the time of immunisation.
--------------------------------------------------------------------------------------------------------------------------------
2 ABDULRAHMAN BASHIR BENGHAZI CHILDREN HOSPITAL
9
25.1.2018
Which of the following is correct regarding human varicella zoster immunoglobulin (VZIG)?
A. Is used to treat severe chicken pox infection
B. Is recommended for all patients with eczema exposed to chickenpox.
C. Is invariably protective against severe varicella.
D. Should be given to a 6 week old baby whose mother has developed chickenpox
E. Should be given to an 18 week pregnant non-immune female who has been exposed to a case of
chicken pox.
Ans:-E
Varicella has a secondary infection rate in household contacts of 90%. It is commonest in spring time, and
the incubation period is 14-21 days. It shares the herpes virus family properties of latency and reactivation
(zoster). Risks to the fetus and neonate relate to the time of infection:
 Less than 20 weeks pregnancy: congenital varicella (limb hypoplasia, microcephaly, cataracts,
growth retardation, skin scarring). High mortality.
 Second to third trimester: herpes zoster in an otherwise healthy infant.
 Minus 7 days to plus 7 days after delivery: severe and even fatal disease (30% mortality). Although
a live attenuated vaccine is available, it is not licensed for use in the UK.
Varicella zoster immunoglobulin is prepared from pooled plasma of UK blood donors with a history of
recent chickenpox or herpes zoster. Being an immunoglobulin, it is a protein concentrate, and should be
stored between 2 and 8oC. Donors are screened for HIV, hepatitis B and hepatitis C. VZIG prophylaxis is
recommended for patients who fulfil all the following criteria:
 A clinical condition that increases the risk of severe varicella, (e.g. immunosuppression, neonates,
pregnant women).
 No antibodies to varicella zoster.
 Significant exposure to chickenpox or herpes.
Severe or fatal varicella can occur despite VZIG prophylaxis. Active immunisation should therefore be
used for susceptible immunosuppressed patients at long term risk. Clinical chickenpox occurs in 50% of
those who receive VZIG prophylaxis, and 10% more will be affected sub-clinically.
Which of the following are true regarding Oral Polio Vaccine (OPV)?
A. It contains 3 strains of live attenuated virus.
B. It is considered ineffective in the presence of diarrhea.
C. It is contraindicated in children on oral steroids 2mg/kg/day
D. Use of the inactive vaccine reduces the carriage of the wild virus.
E. Incidence of vaccine associated poliomyelitis is negligible
Ans:-ABC
Oral polio ('Sabin') vaccine is given by mouth in contrast to the inactivated ('Salk') vaccine (IPV) which is
given IM. Polio vaccines are usually given at two months, three months and four months of age, with a
booster before school usually between 3 and 5 years of age and again, before leaving school, between 15
and 19 years old. Boosters thereafter are not normally necessary, unless travelling to an area where polio is
common, or likely to be exposed to people with polio.
The oral vaccine contains live virus particles which have been attenuated to reduce the risk of neurological
disease. The risk of vaccine associated poliomyelitis is small but not negligible and this together with the
elimination of wild polio from the European Region was the principle reason for the change to the routine
use of IPV in the UK in 2006. Three types of poliomyelitis virus (Types 1, 2 and 3) are included in the
vaccine.
Precaution:-
Postpone if acute illness with pyrexia/diarrhoea/vomiting, Immunodeficiency/treatment with high doses of
steroids/immunosuppresants, First four months of pregnancy
2 ABDULRAHMAN BASHIR BENGHAZI CHILDREN HOSPITAL
10
25.1.2018
Which of the following are true regarding the Meningococcal vaccine which is used in the UK
immunisation schedule?
A. It protects against types B and C.
B. It can cause meningoencephalitis.
C. It is contraindicated in HIV infection.
D. 3 injections are given from 6 months of age.
E. It is a live attenuated vaccine
Ans:-D
Almost all childhood meningococcal disease in the UK is caused by Neisseria meningitidis serogroups B
and C. Meningococcal Group C conjugate vaccine protects only against infection by serogroup C; it can be
given from 2 months of age. After early adulthood the risk of meningococcal disease declines, and
immunisation is not generally recommended after the age of 25 years.
Meningococcal Group C conjugate vaccine provides long-term protection against infection. The
recommended schedule consists of 3 doses given at 3 months, 12 months and as a booster at around 14
years of age.
Side-effects of meningococcal Group C conjugate vaccine include redness, swelling, and pain at the site of
the injection, mild fever, irritability, drowsiness, dizziness, nausea, vomiting, diarrhoea, headache, myalgia,
rash, urticaria, pruritus, malaise, lymphadenopathy, hypotonia, paraesthesia, hypoaesthesia, and syncope.
Hypersensitivity reactions and seizures have been reported rarely. Symptoms of meningism have also been
reported rarely, but there is no evidence that the vaccine causes meningococcal C meningitis. There have
been very rare reports of Stevens-Johnson syndrome. The CSM has advised that vaccination provides
benefit in terms of lives saved and disabilities prevented.
Currently meningococcal vaccine is a purified heat stable extract from the polysaccaride outer capsule of
Neisseria meningitidis.
---------------------------------------------------------------------------------------------------------------------------------
Which of the following are absolute contra-indications to Pertussis immunisation?
A. Epilepsy.
B. Family history of convulsions after immunization.
C. Eczema.
D. Autism.
E. Previous severe local reaction to immunisation
Ans:-All false
Since 2006 acellular pertussis vaccines have been used for routine immunisation in the UK and this change,
together with an assessment of vaccine reactions has led to a significant change in the contraindications and
warnings relating to pertussis immunisation.
The only remaining absolute contraindications for pertussis-containing vaccines are:
• Confirmed anaphylactic reaction to a previous dose of pertussis-containing vaccine, or
• Confirmed anaphylactic reaction to neomycin, streptomycin or polymyxin B (which may be present in
trace amounts in the vaccine).
Of particular note (and in contrast to previous advice), immunisation with pertussis-containing vaccines
should not be withheld despite a previous history of convulsions, high fever, hypotonic-hyporesponsive
episodes (HHE), persistent crying, screaming or severe local reaction following immunisation.
By the time of school entry children in the UK should have received 4 doses of pertussis-containing
vaccines (DTaP/Hib/IPV at ages 2, 3 & 4 months, and DTaP or dTaP at age 3.5 to 4 years (a preschool
booster). The booster dose at school entry was added in 2006 at the same time that the change was made
from whole-cell to acellular pertussis component vaccines.
A boy was born at 26 weeks gestation. He had severe hyaline membrane disease and was ventilated
for 3 weeks. He is now three months old and has just been discharged from hospital. He should
receive pertussis vaccination even if:
2 ABDULRAHMAN BASHIR BENGHAZI CHILDREN HOSPITAL
11
25.1.2018
A. His Apgar scores were 3 at 1, and 5 at 5 minutes.
B. He had a pneumothorax and 4 convulsions in the first week.
C. He was hypoglycaemic and very jittery for a short period.
D. After a pneumothorax, a cranial ultrasound scan showed an intraventricular haemorrhage with some
loss of cerebral substance.
E. He continues to have uncontrolled convulsions two or three times a week
Ans:-ABCD
Children with cerebral damage, a personal history of convulsions, or a family history of febrile convulsions
are at increased risk of a febrile fit following pertussis and measles immunisation. However, these are not
absolute contraindications and they are not at any greater risk of permanent adverse effects from the
vaccines and should receive them. Children whose epilepsy is poorly controlled should not be immunised
until control is achieved, although convulsions themselves are not a contraindication.
The only absolute contraindications for pertussis-containing vaccines are:
• Confirmed anaphylactic reaction to a previous dose of pertussis-containing vaccine, or
• Confirmed anaphylactic reaction to neomycin, streptomycin or polymyxin B (which may be present in
trace amounts in the vaccine).
Contraindication to pertussis immunisation include:-
A. Acute febrile illness
B. Family history of seizures
C. Known cerebral palsy
D. Generalized reaction to previous dose
E. Autism
Ans:-AD
A two-month-old child had his first DTP injection 3 days previously and his mother has returned
because his thigh is swollen. On examination, most of the anterolateral surface of the thigh is red and
indurated. What will you recommend for this child in the future?
A. He should not have any further immunizations
B. He should have all immunisations except pertussis
C. He should receive the acellular pertussis vaccine when next immunized
D. He should receive the oral pertussis vaccine when next immunized
E. He should receive the whole cell pertussis vaccine (as DTP) as usual at his next immunization
Ans:- C
This degree of thigh swelling constitutes a severe local reaction (official definition: 'an extensive area of
redness which becomes indurated and involves most of the anterolateral surface of the thigh or a major part
of the circumference of the upper arm').
In the case of a severe local reaction, the acellular pertussis vaccine should be administered the next time.
This is composed of a selection of proteins from pertussis and is said to have fewer side effects than the
whole cell pertussis vaccine (which is composed of whole killed organism).
A severe generalised reaction is defined as any of the following:
* A temperature >39.5C within 48 hours of pertussis vaccination
* Seizures or encephalopathy within 72 hours of pertussis vaccination
* Prolonged unresponsiveness or collapse
* Anaphylaxis or bronchospasm
* Inconsolable screaming for more than 4 hours within 2 days of immunization.
In the case of a severe generalised reaction, diphtheria and tetanus ONLY should be given next time.
There is no oral pertussis vaccine. Any oral vaccine will be a live vaccine (e.g. oral typhoid, oral polio
(Sabin)).
Childhood immunisation with pertussis is absolutely contraindicated in:
2 ABDULRAHMAN BASHIR BENGHAZI CHILDREN HOSPITAL
12
25.1.2018
A. Immunocompromised patients.
B. Down Syndrome.
C. Cerebral palsy.
D. Family history of convulsion.
E. Rectal temperature of 38.5°C.
Ans:- all false
Comments:
Killed vaccine Contraindications:
Relative: acute illness. Severe local - use acellular vaccine.
Absolute: Severe generalised reaction (fever >39.5 within 48 hours),
bronchospasm,laryngeal oedema,collapse, prolonged
unresponsiveness, prolonged inconsolable crying for more than 4hours
convulsions/encephalopathy within 72 hours.
Which of the following children should not receive live vaccines?
A. Those requiring regular intravenous Immunoglobulin therapy.
B. Those on a prednisolone dose of 3 mg/kg per day for longer than 1 week.
C. Children vvith graft-versus-host disease.
D. Children on chemotherapy.
E. Children who have had a solid organ transplant.
Ans:-ABCDE
---------------------------------------------------------------------------------------------------------------------------------
The following are notifiable diseases:
A. Acquired immune deficiency syndrome (AIDS).
B. Mumps.
C. Tuberculosis (TB).
D. Rubella.
E. Malaria
Ans:-BCDE
Which of the following are indications for influenza vaccination in children?
A. Spastic diplegia.
B. AIDS.
C. Acute lymphoblastic leukaemia.
D. Splenectomy.
E. Bronchiectasis.
Ans:-BCDE
The injectable influenza vaccination is recommended for all children > 6 months old in high risk groups
(chronic lung disease, chronic heart disease, chronic renal disease, immunosuppression (e.g such that
occurs in leukaemia, AIDS and splenectomy) and diabetes). An annual nasal spray influenza vaccine was
introduced into the NHS schedule in 2013 for 2 and 3 year olds. The programme will eventually be
extended to all children between 2 and 16 years of age in the UK.
It is safe for HIV-infected children to have the following immunisations:
A. Measles-mumps-rubella (MMR).
B. Bacille Calmette-Guerin (BCG) .
C. Haemophilus influenzae type B (HIB)
D. Diphtheria-pertussis-tetanus (DPT)
E. Hepatitis B
Ans:-ACDE
2 ABDULRAHMAN BASHIR BENGHAZI CHILDREN HOSPITAL
13
25.1.2018
Current recommendations are that it is safe for HIV-infected children to receive: MMR; oral polio
(inactivated form may be given), pertussis, diphtheria, tetanus, typhoid, cholera, hepatitis B and HIB. They
should not have BCG, yellow fever and oral typhoid.
It is advised by department of Health that children who are HIV positive and who have symptoms of
AIDS can receive the following vaccine:-
A. MMR
B. Hib
C. Pertussis
D. Oral typhoid
E. BCG
Ans:- ABC
Live vaccine:-MMR, Poliomyelitis
Inactivated vaccine:-cholera, Diphtheria, Hib. Hepatitis A &B, Influenza, Meningococcal, Pertussis,
Pneumococcal, Poliomyelitis, Tetanus, Typhoid ( injection),
BCG vaccine, yellow fever, oral typhoid are C/I in symptomatic HIV patient.
A fully immunized 5-year-old has a splenectomy for spherocytosis. Which of the following vaccines
are indicated?
A. Hepatitis B.
B. BCG.
C. Meningococcal group C vaccine.
D. Pneumococcal.
E. Influenza.
Ans:-DE
Vaccinations against pneumococcus should be given before a splenectomy, and yearly vaccinations against
influenza are recommended after a splenectomy.
-----------------------------------------------------------------------------------------------------------------------------
Members of the innate immune system include:
A. T Iymphocytes.
B. antimicrobial peptides.
C. Insulin-like growth factors.
D. Complement C3.
E. Lectin.
Ans:-BD
The innate or non-specific immune system is the first line of defence against pathogen. It has many
components, including mechdnical barriers (skin, mucou secretions) and soluble factors (complement,
mdnnose-binding lectin) as well as neutrophils and macrophages. Anti-microbial peptides, such as beta
defensins are proteins secreted by endothelial cells. Specific immunity are provided by T cells and B cells,
and results in the generation of immune memory, resulting in rapid responses on subsequent challenge by
a given pathogen.
T lymphocytes:-
A. Are derived from a precursor cell in the bone marrow .
B. 95% die in the thymus.
C. All have the alpha-beta heterodimeric T cell receptor.
D. Are usually activated by intact protein antigens .
E. Are prominent in type II hypersensitivity reactions
Ans:- AB
2 ABDULRAHMAN BASHIR BENGHAZI CHILDREN HOSPITAL
14
25.1.2018
T lymphocytes comprise 70-80% of the lymphocyte population and are haematopoietic cells. They are
produced in bone marrow and paracortical regions of lymph nodes and spleen. Maturation occurs in the
thymus, and involves a complicated series of positive and negative selection steps coordinated by the
thymocytes and dendritic cells within the thymus, resulting in apoptosis of unwanted cells. The TCR is a
heterodimer of two chains (alpha and beta, or gamma and delta) linked by disulphide bonds and is found at
the cell surface associated with a complex of other polypeptides known collectively as CD3. The two types
of TCR are called TCR-1 (gamma-delta) and TCR-2 (alpha-beta). TCR-1 cells are thought to have a
restricted repertoire and to be mainly non-MHC restricted. TCR-2 cells recognise peptide fragments
associated with MHC molecules.
The following are known functions of T helper cells :-
A. Production of T cell growth factors such as IL-2
B. Promotion of immunoglobulin isotype class switching
C. Production of perforin
D. Production of suppressive cytokines such as IL-10, TGF-beta
E. Induction of the maturation of immature dendritic cells
Ans:-ABDE
Perforin is produced by cytotoxic T lymphocytes (CTL) to kill target cells.
Helper T lymphocytes (HTL) are usually CD4+ T-cells. HTL can be sub-classified into TH1 and TH2 cells.
More recently, further sub-populations of CD4+ T cells have been identified, termed Th3 and Tr1 cells. In
general, TH-1 cells secrete pro-inflammatory cytokines such as IL-2, IFN-gamma, TNF-alpha , TNF-beta
and are important for cell mediated immunity. They also promote the production of opsonising antibodies.
TH-2 cells secrete IL-4, IL-5 and IL-10 and are important for humoral responses. TH-3 and Tr1 are not yet
well defined but the former secrete predominantly IL-10 while the latter secrete mainly TGF-beta. It must
be remembered that this differential cytokine expression is not absolute.
Which type of T-cells have the major role of killing host cells infected by pathogens
A. TH-1 (CD4).
B. TH-2 (CD4).
C. TC (CD8).
D. TH-1 (CD8).
E. TH-2 (CD8)
Ans: C
T cells are divided into three functional classes:
 TC (CD8) cells kill host cells infected by pathogens—notably viruses—that replicate within the
cytoplasm of host cells;
 TH-1 (CD4) cells activate macrophages and thus allow them to destroy pathogens such as M
tuberculosis and P carinii, which inhabit macrophage vesicles;
 TH-2 (CD4) cells activate B cells to produce antibodies . Thus, T lymphocytes are critical in both
humoral and cell-mediated immunity, and deficiencies can be devastating.
Which type of T-cells have the major role of activating macrophages thus allowing them to destroy
pathogens such as M tuberculosis and P carinii
A. TH1 (CD4)
B. TH2 (CD4)
C. TC (CD8)
D. TH1 (CD8)
E. TH2 (CD8)
Ans:- A
The following contribute to the natural immunity transmitted in breast milk:
A. Macrophages
2 ABDULRAHMAN BASHIR BENGHAZI CHILDREN HOSPITAL
15
25.1.2018
B. natural killer cells.
C. Lysozyme.
D. mast cells.
E. secretory IgA.
Ans:-ABCE
The following HLA associations are correct:
A. HLA DR3 and dermatitis herpetiformis.
B. HLA A3 and schizophrenia.
C. HLA DR2 and Goodpasture’s syndrome.
D. HLA DQW2 and coeliac disease.
E. HLA DR7 and Hashimoto’s thyroiditis
Ans:- ACD
Certain HLA alleles are associated with increased risk of disease. The following are further examples:
A3/B14 in haemochromatosis; A28 in schizophrenia: B5 in Behçet’s disease and ulcerative colitis; Dfl3,7
DQW2 in coeliac disease; B27 in ankylosing spondylitis and psoriatic arthropathy: DR2 in Goodpasture’s
syndrome and narcolepsy; DR3 in chronic active hepatitis, Grave’s disease, myasthenia gravis, Addison’s
disease and Sjôgren’s syndrome: DR4 in rheumatoid arthritis and insulin dependent diabetes, and DR5 in
Hashimoto’s thyroiditis.
The following statements are true regarding superantigens:
A. They are very strongly antigenic bacterial polysaccharides.
B. They bind to B cell receptors.
C. Lead to excess stimulation of T cell proliferation and interleukin-2 production.
D. They are responsible for gram-negative toxic shock.
E. Lead to stimulation of production of TNFa and other cytokines.
Ans:-CE
Superantigens are an unusual type of bacterial toxin comprising of proteins which exert their effect by
forming a bridge between the MHC complex of antigen-presenting cells and T cell receptors. Because the
binding to T cell receptors is indiscriminate, excess stimulation of T cell proliferation occurs leading to
abnormally high levels of IL-2,with symptoms of nausea, vomiting, fever and malaise. They have a role in
Gram-positive septic shock, particularly toxic shock syndrome and staphylococcal food-borne disease. It is
possible that they may also have a role in the pathogenesis and clinical features of Kawasaki disease.
Which of the following regarding the Fc component of antibody are true?
A. Determines antigen bindinq capecificity.
B. Determines the ability to cross the placenta.
C. Determines the ability to bind with mast cells.
D. When produced in excess, excreted in urine as Bence-Jones protein.
E. Determines metabolic half life -of the whole molecule
Ans:-BCE
The Fc component of antibody composed of two heavy chain and it bind to various cell receptors and
complement proteins. It mediate different physiological effects of antibodies ( opsonization, cell lysis, mast
cell, basophil and eosinophils degranulation and other process. Bence –Jones protein are free
immunoglobulin light chains.
The secondary immunoglobulin response is not characterized by:-
A. Isotype switching to IgG
B. Production of immunoglobulin with higher affinity to the antigen
C. Production of memory plasma cells
D. A quantitatively increased antibody production as compared to the primary immune response
E. Requirement of help from T-cells
2 ABDULRAHMAN BASHIR BENGHAZI CHILDREN HOSPITAL
16
25.1.2018
Ans:- C
Memory B-cells are responsible for the secondary Ig response. Memory B-cells differ qualitatively as well
as quantitatively from naïve B cells. After priming during the primary immune response, the frequency of
antigen-specific B-cells may increase 10-100 fold.
Therefore the secondary immune response to a specific antigen occurs more rapidly than the primary
response, and larger amounts of antibody are produced. Memory B-cells also have higher affinity for
antigen as they have undergone affinity maturation in the germinal centres of lymphoid tissue. Memory B-
lymphocytes have undergone class switch and produce mainly IgG but also other immunoglobulin isotypes.
Both primary and secondary immunoglobulin responses require T-cell help. (NB. Certain bacterial antigens
can activate B cells directly, e.g. LPS). Plasma cells are the effector cells of both primary and secondary
immune responses. They secrete large amounts of antibody, but are short lived cells in order to limit
antibody responses.
IgA:-
A. is found in breast milk.
B. is important in mucosal immunity.
C. can fix complement.
D. is the most abundant immunoglobulin in the blood.
E. is deficit in coeliac disease.
Ans:-ABE
IgA:
A. Is present in breast milk and saliva.
B. Isolated deficiency is very serious.
C. Can fix complement.
D. Is the most abundant in blood.
E. Has two subclasses and secretory component.
Ans:-AE
The following immunoglobulin are responsible for:
A. IgA leads to bacterial agglutination.
B. IgD is involved in inhibition of viral replications.
C. IgE : Type 1 sensitivity reaction
D. IgG : neutralisation of bacterial toxins
E. IgM : complement activation.
Ans:-ACDE
The following statements are correct:
A. IgA can activate complement via the alternative pathway.
B. IgA normally represents less than 1% of the serum immunoglobulin pool.
C. J-chains are associated with both lgA and gM.
D. the Fab fragment determines the class of immunogiobulin.
E. IgE can cross the placenta.
Ans:- AC
Secretory IgA is dimeric. the two subunits joined by a J-chain; IgM is pentameric. IgA and IgE can fix
complement via the alternative pathway: IgG and IgM via the classical pathway. Cf the normal total
immuncglobulin pool IgG makes up approx 75%, lgA 15% arid gM 10%. There are traces of IgD and IgE.
Only lgG can cross the placenta. The constant region of the molecule (Fc) determines the class of
immunoglobulin. The Fab region has a variable (V) and constant (OH) domain, the former being unique for
each antibody.
Immunoglobuline G ;
A. Has MW of 900 000 dalton
B. Is the first immunoglobuline class to raise in acute infection
2 ABDULRAHMAN BASHIR BENGHAZI CHILDREN HOSPITAL
17
25.1.2018
C. Has half life of 21 days.
D. Fall in the first few weeks of life.
E. Is the main immunoglobuline class involved in the ABO blood group system
Ans;CD
Which of the following statement regarding immunoglobulin classes and function are Correct
A. IgM is pentamer.
B. IgA is always dimmer.
C. IgD is T cell receptor.
D. IgG4 is responsible for polysaccharide response.
E. All IgG classes are involved in complement fixation pathway
Ans ; A
Passage of IgG through the placenta is determined by:-
A. Molecular weight.
B. Fc fragment.
C. JA chain.
D. Secretary complement
Ans:- B
Which statement about immunoglobulin is false
A. IgG comprise 70-80% of the total.
B. IgG & IgM can both cross the placenta in human.
C. The half life of IgG is about 25 days.
D. The spleen seems to play an important role in producing IgM antibody.
E. IgA comprise 10-15% of the total and is found mainly in the secretion
Ans ;B
C reactive protein ;
A. Is synthesized in the liver.
B. Production is stimulated by interleukin-6.
C. Freely crosses the placenta.
D. Concentration raises following intraventricular hemorrhage in premature infant.
E. Usually take more than week to return to normal level following clinical resolution of bacterial
infection in the newborn
Ans ;AB
A 6-year-old boy with a sore throat, cough, and fever is noted to have a marked elevation in his
serum C-reactive protein. This finding indicates which of the following?
A. Developing autoimmune reaction.
B. Ineffective immune response
C. Meningitis
D. Non-specific inflammation
E. Respiratory compromise
Ans:-D
C-reactive protein is one of the most commonly measured acute-phase reactants, which are a group of
serum proteins showing a rapid increase in serum concentration in response to any inflammatory process.
This finding is entirely non-specific & ndash; it only indicates a recent inflammatory process.
An autoimmune reaction (choice A), which is certainly a concern with streptococcal pharyngitis, is
suggested by the development of a rising ASO (antistreptolysin O) titer weeks after the illness. Acute phase
reactants are not specific to autoimmune processes.
Increases in C-reactive protein indicate a healthy immune response to an infective pathogen. An ineffective
immune response (choice B) would not elicit acute-phase reactions.
Meningitis (choice C) cannot be diagnosed from a blood test. The diagnosis of meningitis requires the
appropriate clinical signs (neck stiffness, mental status changes) and a positive lumbar puncture.
2 ABDULRAHMAN BASHIR BENGHAZI CHILDREN HOSPITAL
18
25.1.2018
Respiratory compromise (choice E) produces changes in arterial blood gases and blood pH. Acute-phase
reactants do not reflect respiratory status.
Healthy term neonate differ from an adult in the following ways ;
A. Less complement.
B. Decreased IgG level.
C. Fewer B lymphocytes.
D. Lower level of secretory IgA.
E. Higher level of C reactive protein
Ans;AD
The following are opsonine;
A. IgD
B. Complement
C. C reactive protein
D. Interferone-ά
E. IgG
Ans; BCE
The following may indicate an underlying specific immunoglobulin deficiency:
A. Recurrent oral thrush.
B. Recurrent tonsillitis.
C. Recurrent viral gastroenteritis.
D. Recurrent conjunctivitis.
E. Severe aphthous ulceration.
Ans:-BC
Oral thrush suggests cell-mediated defect. IgA deficiency or IgG sub-class deficiency may result in
recurrent tonsillitis and viral gastroenteritis. In addition to abscesses and lymphadenopathy, mucus
membrane infections such as conjunctivitis, rhinitis and stomatitis have been reported in chronic
granulomatous disease.
Oral ulceration may be:
Common: aphthos, traumatic, hand, foot and mouth disease, herpangina, chemical burns (alkali, acid,
aspirin).
Uncommon: neutrophil defects (agranulocytosis, leukaemia, cyclical neutropenia), SLE, Behcet
Syndrome, necrotising ulcerative gingivostomatitis, syphilis, Crohn's Disease, histoplasmosis.
Conjunctivitis is common, since the conjunctivae react to a wide range of bacterial and viral agents,
allergens, irritants, toxins, and they are inflamed in systemic diseases.
Which of the following is true concerning immunity to viruses?
A. IgA can offer protection at mucosal surfaces.
B. Cytotoxic T cells are activated before natural killer cells during the course of infection.
C. Viruses stimulate the non-immune cells that they infect to produce interferon-γ.
D. Non-enveloped viruses are susceptible to damage by complement.
E. Influenza virus can avoid antibody recognition by mutational changes in its nucleocapsid proteins.
Ans:-A
Natural killer cells are activated faster than cytotoxic T cells. Infected non-immune cells produce
interferon-α and – β , whereas interferon-γ is produced by T cells. Influenza virus mutates its surface
neuraminidase and haemagglutinin to avoid antibody recognition. Enveloped viruses are susceptible to
complement attack
Which of the organs below is considered an immunologically privileged site :-
2 ABDULRAHMAN BASHIR BENGHAZI CHILDREN HOSPITAL
19
25.1.2018
A. skin
B. thymus
C. bone marrow
D. testicle
E. pancrease
Ans:- D
Some tissues fail to elicit an immune response and are termed immunologically privileged sites- brain,
cornea, testis, uterus and arguably liver. Several factors may contribute to immune privilege.
The blood-brain barrier may block entry of lymphocytes into this organ and the avascularity of the cornea
may also limit antigen/lymphocyte interaction.
The eye also has high concentrations of TGF-beta which is a cytokine with immunosuppressive properties.
Both the eye and the testis constitutively express Fas ligand. This membrane protein can bind to Fas on
infiltrating leukocytes, and can induce apoptotic cell death in the Fas expressing cell.
Liver transplantation may induce antigen-specific tolerogenic effects. The precise mechanisms are not
clear, but have been attributed to the establishment of micro-chimerism, the existence of tolerogenic donor
immature dendritic cells or their precursors within the liver graft.
In contrast, skin is highly immunogenic. Until modern HLA typing techniques developed, skin grafting was
used as a clinical test before solid organ transplantation to determine the tissue type compatibility between
the donor and recipient.
A patient has secondary antibody deficiency due to B-cell suppression. His primary pathology is
likely to be
A. Waldenstrom macroglobulinemia.
B. X-linked agammaglobulinemia.
C. immunoglobulin A deficiency.
D. immunoglobulin M deficiency.
E. common variable immunodeficiency
Ans: A
Antibody immunodeficiency can be primary or secondary.
Secondary antibody deficiency due to B-cell suppression can occur with multiple myeloma, Waldenstrom
macroglobulinemia, or chronic lymphocytic leukemia. Secondary deficiencies leave patients susceptible to
the same pathogens that can cause recurrent pneumonia in patients with primary antibody deficiencies
Primary antibody deficiencies include X-linked agammaglobulinemia, common variable immune-
deficiency, selective immunoglobulin A or immunoglobulin M deficiency, and hyperimmunoglobulin M
immunodeficiency.
These disorders are characterized by chronic or recurrent pyogenic infection, especially pneumonia, caused
by encapsulated bacteria (eg, S pneumoniae, H influenzae, and S aureus) and P aeruginosa . Untreated or
recurrent pneumonia may lead to bronchiectasis.
Regarding the early immunological development of the infant;
A. In the newborn , there is significant quantity of circulating IgM
B. IgG is actively transported across the placenta
C. In the normal newborn , neutrophils function is reduced
D. Large thymus gland detected on chest X ray is normal finding
E. Circulating immunoglobuline level will have reached normal adult level by 6 month
Ans ; BCD
Which of the following statements is true concerning immunity to bacteria?
A. Antibodies to secreted bacterial products play no protective role.
B. Bacteria opsonised by antibodies and complement are more effectively phagocytosed than those
opsonised by antibodies alone .
2 ABDULRAHMAN BASHIR BENGHAZI CHILDREN HOSPITAL
20
25.1.2018
C. Humoral rather than cellular immunity is predominant in protection against all types of bacteria.
D. Phagocytes cannot engulf bacteria in the absence of antibodies .
E. Endotoxin induces shock mainly through the activation of T cells .
Ans:-B
Antibodies such as those to cholera, diphtheria and tetanus toxin can play a major role in protective
immunity in these infections. Cellular immunity is essential in protection against intracellular bacteria, eg
mycobacteria. Phagocytes interact directly but weakly with bacteria, or strongly if they are complement-
opsonised. Endotoxin activates macrophages by binding to CD14.
The following the need for investigation of immune function ;
A. Several episodes of URTI and GE in the space of one year
B. Two episodes of bacterial meningitis
C. Absence of tonsil in the second year of life
D. History of chronic diarrhea and oral candidiasis
E. History of first cousin marriage in the parents of child with recurrent pneumonia
Ans; BCDE
The following infection are appropriately paired with relevant immune deficiency;
A. Recurrent pneumococcal infection and complement deficiency
B. Recurrent viral infection and hypogammaglobulinemia
C. Cutaneous abscess and Ig A deficiency
D. Osteomyelitis and CGD
E. Disseminated viral infection and T – cell deficiency
Ans;ADE
the following statement concerning immunity are correct ;
A. decreased cellular immunity may lead to increased susceptibility to fungal Infection.
B. hypogammaglobulinemia may lead to increased susceptibility to infection gram positive cocci
C. IgG level are usually high from the age of 3 -12 month
D. decreased cellular immunity has recognized association with hypoparathyroidism
E. the immunologic defect in chronic granulomatous disease is reduced efficacy of gammglobuline
Ans ;ABD
Which of the following definitely excludes antibody deficiency?
A. Normal serum immunoglobulins.
B. Good IgG antibody responses to immunisations.
C. The presence of existing antibody responses to past infections.
D. Normal IgG subclasses.
E. Normal peripheral blood lymphocyte subpopulations
Ans:-B
Normal immunoglobulins, including subclasses, do not exclude antibody deficiency. Hence in patients with
good history of recurrent (proven) bacterial infections, responses to Haemophilus influenzae,
Pneumococcus spp. and tetanus toxoid should all be assessed, as should postimmunisation responses if
required. Antibodies to past infections and haemagglutinins can be helpful in assessing a patient.
Of the following options, the best initial test for a workup for immunodeficiency is:
A. Serum levels of IgG, IgM, IgA, IgE.
B. A complete blood count with differential.
C. Peripheral T-cell phenotyping.
D. Erythrocyte sedimentation rate.
E. Postimmunization immunoglobulin levels
Ans:-B
The initial step in the workup of a child with a suspected immunodeficiency should always include a
complete blood count and differential, as this test will reveal disorders involving lymphopenia (particularly
2 ABDULRAHMAN BASHIR BENGHAZI CHILDREN HOSPITAL
21
25.1.2018
T-cell lymphopenias) and neutropenia. Other hints can be found by the CBC, including
small platelets associated with the Wiskott–Aldrich syndrome, anemias, and evidence of eosinophilia.
An erythrocyte sedimentation rate is a nonspecific marker of inflammation and would not point to a
specific immunodeficiency. While quantitative immunoglobulins, peripheral T-cell phenotyping, and lack
of antibody response to standard immunizations would all be part of an immunodeficiency investigation,
these would not be considered first steps in the evaluation.
The following are general feature of primary immunodeficiencies ( PID );
A. The overall incidence is very similar to that of phenylketonuria
B. Failure to thrive is commonly associated feature.
C. Bone marrow transplantation is indicated in all T & B lymphocyte function deficiency.
D. PID should be suspected in any child who has had two ( or more ) invasive bacterial infections.
E. PID commonly present with infection of increased severity
Ans:-ABDE
The following statement(s) concerning inherited immunodeficiency syndromes are true:
A. The commonest inherited form of severe combined immune deficiency is x-linked
B. The incidence of lymphoid malignancy is increased in ataxia telangiectasia
C. The defect in Chediak-Higashi syndrome is in T-cell activation
D. Deficiency in glucose-6-phosphate dehydrogenase results in impaired motility of leukocytes
E. The molecular defect in X-linked hyper IgM syndrome is in the expression of CD40 on B-cells
Ans:-AB
Several defects can lead to the phenotype of severe combined immunodeficiency, the commonest
inherited form is X-linked. There is a mutation in the gene encoding the common gamma chain – a chain
shared by receptors for a number of interleukin cytokines. There is thus impaired T-cell differentiation as
the interleukins are important T-cell growth and differentiation factors. Humoral immunodeficiency
occurs because of a lack of T-cell help. The commonest autosomally inherited forms are adenosine
deaminase and purine nucleotide phosphorylase deficiency. ADA and PNP are enzymes which catalyze
the metabolism of purines via the uric acid pathway. Deficiency of either of these enzymes leads to the
accumulation of toxic metabolites which severely affect lymphocyte production and function. Other
known genetic defects that lead to severe immunodeficiency include mutations of DNA-dependent
protein kinase (DNA-PK) and recombination activating genes (RAG). Lack of expression of MHC class
II or MHC class I molecules also cause severe immunodeficiency. Ataxia telangiectasia is due to defects
in ATM, a protein that is important in DNA repair. Patients have impaired cell mediated immunity, low
IgE, IgA and IgG2 and are prone to respiratory infections and skin infections. Lymphoid malignancies as
well as carcinomas occur with with increased frequency in these patients. Chediak-Higashi syndrome is
an autosomal recessive disorder, characterised by giant granules in phagocytes due to a genetic defect in
intracellular vesicle formation and hence impaired intracellular killing following phagocytosis. Other
clinical features include partial oculocutaneous albinism and abnormal platelet function. Cytolytic T-cell-
mediated killing is normal, but NK killing is impaired. G6PDH is an important enzyme in the hexose
monophosphate shunt. This pathway maintains glutathione in a reduced state protecting the red cell
membrane from oxidant stress . The enzyme also functions in the respiratory burst - a process important
for intracellular killing by phagocytes. Complete G6PDH deficiency results in susceptibility to chronic
bacterial and fungal infection - the clinical picture resembles a mild form of chronic granulomatous
disease.
X-linked hyper IgM syndrome is due to the lack of CD40 ligand (CD40L) expression on activated T-
cells. This leads to impairment in immunoglobulin isotype switching and germinal centre formation as
CD40/CD40L interaction is needed for these processes. Serum IgM levels are high with deficiency of
IgG and IgA. Clinically, these patients are particularly susceptible to opportunistic lung pathogens such
as Pneumocystis carinii.
In primary immune deficiency:-
2 ABDULRAHMAN BASHIR BENGHAZI CHILDREN HOSPITAL
22
25.1.2018
A. In chronic granulomatous disease the infection is usually due to catalase producing bacteria.
B. Severe combined immunodeficiency is an autosomal dominant disorder.
C. In DiGeorge syndrome hypoadrenalism is part of the disorder.
D. Bone marrow transplant is. the treatment of choice for Chediak-Higashi syndrome.
E. Thrombocytosis is part of Wiskott Aldrich syndrome.
Ans:-AD
The following is true regarding therapy for primary immunodeficiencies ( PIDs ) ;
A. gene therapy is indicated for common variable immunodeficiencies
B. gamma – interferon can be useful in chronic granulomatous disease
C. C1 esterase inhibitor concentrate for hereditary angioedema
D. Bone marrow transplantation is indicated for severe combined immunodeficiency
E. Immunoglobuline replacement is indicated for hyper IgM syndrome
Ans ; BCDE
Definitive prenatal diagnosis is possible for the following the primary immunodeficiencie
A. Selective Ig A deficiency.
B. Wiskott Aldrich syndrome
C. X linked form of severe combined immunodeficiency.
D. DiGeorge syndrome
E. Common variable immunodeficiency
Ans ; BCD
During the last trimester, IgG is actively transported across the placenta to suppy passive immunity
to the fetus. Which disease occurring during pregnancy is most likely to lead to the neonate having
low immunoglobuline levels and hence being prone to bacterial infections.
A. Intestinal lymphangectasia.
B. Systemic lupus erythematosus.
C. Myasthenia gravis.
D. Ulcerative colitis.
E. Prematurity.
Ans:-A
The following statements on cellular immunity are correct
A. cytotoxic T cells (CTL) carry +8 glycoprotein and respond to peptides presented by HLA class 1
B. CTL produce interleukin 2 (IL-2).
C. Di George syndrome is associated with impaired cellular immunity.
D. Lymphopenia is a feature of ataxia telangiectasia.
E. Wiscott—Aldrich disease is an autosomal recessive condition characterised by depressed cellular
immunity
Ans:- ACD
T helper cells (Th) express 4+ and recognise HLA (major histocompatibility complex) class 2. Most CTL
are 8+ and restricted to class 1 HLA. A sub-class of Th cells produce IL-2. Di George syndrome (thymic
aplasia) is associated with impaired T cell production. Immunoglobulin levels are usually adequate and
there is an increased risk of autoimmune disease. Ataxia telangiectasia has several deficiencies of
immunoglobulins and lymphopenia: it is autosomal recessive. Wiscott—Aldrich syndrome is X-linked, the
gene defect leading to abnormalities in the regulation of sialophorin (0043). The condition is characterised
by thrombocytopenia and lymphopenia.
A 1-year-old boy has a history of multiple episodes of otitis media, sinusitis, and pneumonia. He has
not had any fungal, protozoan, or mycobacterial infections. You suspect a disorder of humoral
immunity. Which of the following is the best initial screening test?
A. Dihydrorhodamine (DHR) flow cytometry test.
B. Quantitative serum immunoglobulins.
C. Examination of the peripheral blood smear.
2 ABDULRAHMAN BASHIR BENGHAZI CHILDREN HOSPITAL
23
25.1.2018
D. Total hemolytic complement (CH50).
E. Anergy panel
Ans:-B
Most primary immunodeficiency diseases involve abnormalities of immunoglobulin concentration or
function (humoral immunity). The hallmark of immunoglobulin deficiency is increased susceptibility to
sinopulmonary infections caused by encapsulated bacteria. Serum immunoglobulin concentrations should
be measured in such patients. The dihydrorhodamine (DHR) flow cytometry test is used to diagnose
chronic granulomatous disease, a disorder of phagocyte oxidative metabolism. Examination of the
peripheral smear can yield valuable information about neutropenia, lymphopenia, or thrombocytopenia,
which is useful and adjunctive but does not provide a definitive diagnosis. Total hemolytic complement
assays are useful to exclude isolated deficiencies of complement components. Sinopulmonary infections are
generally not associated with complement deficiency. Anergy panels are used to evaluate cell-mediated
immune function.
Which of the following pathogens are associated with the accompanying immune disorder?
A. Mycobacteria and type 1 cytokine defects.
B. Gram-negative bacteria and complement deficiency.
C. Enterovirus and antibody defects
D. Staphylococcus and complement deficiency.
E. Meningococcus and neutrophil defect.
Ans:-AC
The pathogens associated with an underlying immune disorder are:
• mycobacteria- type 1 cytokine defects
• Gram-negative bacteria- neutrophil defect.
• enterovirus- antibody defects as well as defects in cell –mediated immunity
• Staphylococcus - neutrophil defect
• Meningococcus - complement deficiency
• Pneumococcus and Haemophiius influenzae B – antibody and complement defects
• Salmonella- type 1 cytokine defect-, and Cell mediated defects.
• Mycoplasma- antibody defects
• herpes viruses- defects in cell-rnediated immunity.
--------------------------------------------------------------------------------------------------------------------------------
The following is true of X – linked agammaglobulinemia ( Brutons disease ) ;
A. Plasma cells are absent from bone marrow.
B. it seldom presents before the age of 5 years.
C. it commonly present with infection of the lungs and sinuses .
D. the underling defect is an abnormality of the gene for B cell specific tyrosine kinase ( BTK )
E. early replacement immunoglobuline therapy can help to prevent chronic lung damage
Ans ;ACDE
A 3-year-old boy presented with recurrent attacks of pneumonia and otitis media since the 1st
birthday. Your diagnosis is X-linked agammaglobulinemia. of the following, the MOST likely
offending organismis
A. CMV.
B. Mycoplasma.
C. Pneumocystis jiroveci.
D. Staphylococcus aureus.
E. Streptococcus pneumonia.
Ans:-E
2 ABDULRAHMAN BASHIR BENGHAZI CHILDREN HOSPITAL
24
25.1.2018
A 4.5-year-old boy presented with history of recurrent attack of secretary otitis media and purulent
nasal discharge, Haemophilus influenza revealed by culture of ear discharge, on physical
examination there is no tonsillar tissue and no palpable lymph nodes. Of the following, the MOST
appropriate test to confirm the diagnosis is measurement of
A. flow cytometry.
B. lgA concenteration.
C. isohemagglutinins titer.
D. IgG and lgM concenteration.
E. antibodies to antigens of routine immunizations
Ans:-A
The diagnosis of X-linked agammaglobulinemia (XLA) should be suspected if lymphoid hypoplasia is
found on physical examination (minimal or no tonsillar tissue and no palpable lymph nodes). Flow
cytometry is an important test to demonstrate the absence of circulating B cells, which will distinguish this
disorder from common variable immunodeficiency, the hyper-lgM syndrome, and transient
hypogammaglobuiinemia of infancy.
In X-linked agammaglobulinaemia all of the following are true except:
A. B cells are absent.
B. It is X-linked recessive and is caused by defects in a gene located at Xq22.
C. The most common organisms causing infection are Staphylococcus aureus and N. meningitidis.
D. Infection with Giardia lamblia is not uncommon.
E. Neutropenia may be present, particularly during severe infections.
Ans:-C
This affects males, with a repeated history of bacterial infection. It usually occurs after 6 months of age and
before 2 years of age. Mild forms may present later in childhood, or even in adolescence or early
adulthood. The most frequent sites of infection are otitis, sinusitis and pneumonia, and the most common
organisms are Haemophilus influenzae and Streptococcus pneumoniae. Non-infectious manifestations (e.g.
arthritis) are unusual in XLA, but have been described. Pneumocystis carinii pneumonia is very rare, but
infection with Giardia lamblia is not uncommon. Neutropenia may be present during severe infections, but
not other times. All isotypes of immunoglobulins are absent or very low. Circulating mature B cells are
absent, as are specific antibodies. T cells are normal. Children with XLA will usually have complications,
e.g. bronchiectasis, deafness, enteroviral meningo-encephalitis and neutropenia.
The treatment is lifelong immunoglobulin replacement every 6–8 weeks and maintenance of IgG to the
level of IgG at 8 g/l. Infections should be treated, and physiotherapy should be given for bronchiectasis.
Referral should be made to a geneticist for all boys with XLA, as there will be risk for their offspring. All
of their daughters will be carriers, but sons will be normal.
A couple brings their son in to a specialty clinic for evaluation of recurrent bacterial infections
involving the respiratory tract. Other family members have a similar disorder, as noted in the
pedigree above. Which of the following is the most likely diagnosis?
A. Bruton's agammaglobulinemia.
B. Common variable immunodeficiency.
C. DiGeorge syndrome.
D. Hereditary angioedema.
E. Isolated IgA deficiency
Ans:-A
Several immunodeficiency disorders have X-linked genetics, including Bruton's agammaglobulinemia,
Wiskott-Aldrich syndrome, and some cases of severe combined immunodeficiency diseases. Bruton's
agammaglobulinemia is characterized by recurrent respiratory infections caused by pyogenic organisms.
Common variable immunodeficiency (choice B) is a relatively common, but probably heterogeneous, group
of acquired and familial diseases. It is distinct from X-linked (Bruton's) agammaglobulinemia.
2 ABDULRAHMAN BASHIR BENGHAZI CHILDREN HOSPITAL
25
25.1.2018
DiGeorge syndrome (choice C) is due to a developmental malformation of the third and fourth pharyngeal
pouches leading to failure of the thymus, and sometimes the parathyroids, to develop. Defective cellular
immunity and abnormalities of calcium metabolism are typical.
Hereditary angioedema (choice D) is a usually recessive genetic disease caused by deficiency of C1
esterase inhibitor.
Isolated IgA deficiency (choice E) can be acquired or genetic, but is not usually X-linked.
An 18 month old male presents with recurrent pneumonia ear infections and tonsillitis.. The family
history includes a previous male infant death at 2 years of age from pnumococcal meningitis. The
parents are therefor verv concerned and wish further investigations to be carried out. After the
result have returned, it is found that the child has absent or decreased level of immunoglobulins A,
G and M. He has reduced response to blood group antibodies to immunisations and an increased
percentage of E rosette with red blood cells. His PHA and nitrobiue tetrazolium (NBT) test are
norma1 . The ultimate disorder is X-linked agammag!obulinemia. Which of the following are
appropriate treatments (more than one answer may be given)?
A. A course of benzylpenicillin for an acute pneumonia and tonsillitis.
B. Azithromycin for 3 days every fortnight as aprophylctic antibiotic
C. Intravenous immunoglobulin.
D. A booster vaccine to DPT, Hib and meningitis C.
E. Bone marrow transplantation.
Ans:-BCDE
Prenatel diagnosis is possible. Investigations show a profound failure of B cell d evelopment so plasma
cells from the bone marrow are absent. There is an abnormal B-cell function and number so there are
isohaemagglutinins or surface immunoglobulins. Investigations undertaken include those stated. Including
finding an increase in OKT3 (49-63% of cases). Neutrophil, lymphocyt, PHA., nitroblue tetrazolium
(NBT), sweat test and T-cell function are all normal. The PHA test measures T-cell proliferation. The T-
cell lymphocyte s are identified by their ability to form rosettes with sheep red blood cells.T-ceII
proliferation gives the total percentage of lymphocytes; the number is increased if no B-cells are present.
Monoclonal antibodies are now used to identify and characterize T-cells. Monoclonal antibodies against
T3 receptors measure the total Tcell number, T4 receptors, the T helper Cell population and T8 receptors
in the T suppressor/cytotoxic (Ts/c) group. Treatment includes prophylactic antibiotics and intravenous
immunoglobulins. Bone marrow transplantation is carried out if medical intervention fail.
A 7-month-old male presents to your office with chronic enteroviral infection and failure to thrive.
The mother is very concerned as multiple family members have died of infection in infancy in the
past. A complete blood count demonstrates a normal total white blood cell count, but further
investigation with flow cytometry detects normal numbers of T cells and the absence of any
circulating B cells. The patient’s diagnosis is most likely to be which of the following?
A. Severe combined immunodeficiency.
B. 22q11 syndrome.
C. X-linked agammaglobulinemia.
D. Adenosine deaminase deficiency.
E. Vertically transmitted HIV infection
Ans:-C
Normal T-cell numbers but absent B cells indicate a primary disorder of the humoral immune system
leading to B-cell maturation arrest. The most common of these is X-linked agammaglobulinemia, which
accounts for over 80% of these disorders. The significant family history of early death from infection is a
clue that this patient’s disorder may be genetic in origin. XLA is caused by an abnormality in Bruton’s
tyrosine kinase resulting in a block preventing maturation past the CD19+ pro-B cell stage. The lack of B
cells in the periphery leads to susceptibility with typical organisms dependent on humoral immune system
2 ABDULRAHMAN BASHIR BENGHAZI CHILDREN HOSPITAL
26
25.1.2018
function. The other listed disorders all result in T-cell abnormalities, which ultimately cause
abnormalities in B-cell function, but cause disorders in both cellular and humoral immune system function.
Which of the following definitely excludes antibody deficiency?
A. Normal serum immunoglobulins.
B. Good IgG antibody responses to immunisations.
C. The presence of existing antibody responses to past infections.
D. Normal IgG subclasses.
E. Normal peripheral blood lymphocyte subpopulations
Ans:-B
Normal immunoglobulins, including subclasses, do not exclude antibody deficiency. Hence in patients with
a good history of recurrent (proven) bacterial infections, responses to Haemophilus influenzae,
Pneumococcus spp. and tetanus toxoid should all be assessed, as should postimmunisation responses if
required. Antibodies to past infections and haemagglutinins can be helpful in assessing a patient
Which of the following isn't highly suspicious of immunodeficiency
A. Chronic sinopulmonary infection
B. Continual infection without period of good health
C. Unusual infecting agent
D. Incomplete response to treatment
E. Eczema
Ans ;E
A young mother brings her 2 year old son to the doctor and comments that he is always sick. Upon
further questioning, his mother is adament he was healthy until he was about 9 months old when the
problems began. He has had recurrent respiratory infections due to pneumoccocus and Haemophilus
influenzae. He also has recurrent bouts of ear infections and pharyngitis. His serum concentrations of
immunoglobulins are in the third percentile. He is eventually diagnosed with X-linked
agammaglobulinemia of Bruton. This is a problem with B lymphocytes. Which of the following is a
true feature of B lymphocytes?
A. their immunocompetence is gained in the thymus
B. they are found in the deep cortical region of lymph nodes
C. presence of surface immunoglobuin
D. forms rosette with sheep red blood cells
E. they make up 70% of the lymphocytes in the circulation
Ans:- C
Surface immunoglobulins are seen with B lymphocytes. All of the other characteristics listed (choice a, b,
d, and e) are characteristics of T lymphocytes.Choice (a) is incorrect because B lymphocytes do not become
immunocompetent in the thymus; they gain immunocontpetence in the bursa equivalent (postulated to be
the bone marrow).B lymphocytes are located in the germinal centers and the medullary cords of a lymph
node, therefore choice (b) is incorrect. B lymphocytes do no form rosettes with sheep red blood cells, T
lymphocytes do. B lymphocytes comprise 30% of the circulating lymphocytes, therefore choice (e) is
incorrect.
Patients with disorders of B-cell development are susceptible to infections from encapsulated
organisms and enteroviral infections. These patients typically become symptomatic at what age?
A. In the second decade of life.
B. Between the ages of 5 and 10.
C. In the first three months of life.
D. Between the first 6 and 12 months of life.
E. In the immediate postpartum period.
Ans:-D
2 ABDULRAHMAN BASHIR BENGHAZI CHILDREN HOSPITAL
27
25.1.2018
Disorders of B-cell development result in maturation arrest of B cells and lack of mature B cells in the
peripheral circulation. This condition results in agammaglobulinemia. The affected child depends on
maternal immunoglobulin for humoral immune system protection; however once maternal immunoglobulin
decays (typically by 6 months of age), the child has no protection against infections with encapsulated
bacteria or viruses, such as enterovirus, leading to chronic recurrent infection
Hypogammglobulinemia is manifested by all of the following except :
A. Recurrent otitis media, pneumonia and meningitis are the most common infection
B. Pneumococcus and Haemophilus influenza are the most common infecting agent
C. Patient are often clinically well between episodes of infection.
D. Infant with X-linked agammaglobulinemia are at increased risk of infection with vaccine induced
polio and echovirus encephalitis.
E. Hypogammaglobulinemia is present when the total serum immunoglobuline are < 250 mg/dl
Ans ; C
A child has a history of recurrent infections with organisms having polysaccharide antigens (i.e.,
Streptococcus pneumoniae and Hemophilus influenzae). This susceptibility can be explained by a
deficiency of:-
A. C3 nephritic factor
B. C5
C. IgG subclass 2
D. myeloperoxidase in phagocytic cells
E. secretory IgA.
Ans:-C
IgG is the predominant antibody in the secondary immune response. IgG subclass 2 is directed against
polysaccharide antigens and is involved in the host defense against encapsulated bacteria.
C3 nephritic factor (choice A) is an IgG autoantibody that binds to C3 convertase, making it resistant to
inactivation. This leads to persistently low serum complement levels and is associated with Type II
membranoproliferative glomerulonephritis.
C5 (choice B) is a component of the complement system. C5a is an anaphylatoxin that effects
vasodilatation in acute inflammation. It is also chemotactic for neutrophils and monocytes and increases the
expression of adhesion molecules. A deficiency of C5a would affect the acute inflammatory response
against any microorganism or foreign substance.
Myeloperoxidase in phagocytic cells (choice D) is an element of the oxygen-dependent pathway present in
phagocytic cells that effectively kills bacterial cells. The hydrogen peroxide-halide complex is considered
the most efficient bactericidal system in neutrophils. Chronic granulomatous disease is associated with a
deficiency of NADPH oxidase, which converts molecular oxygen to superoxide (the first step in the
myeloperoxidase system). Patients are susceptible to granulomatous infections and staphylococcal
infections.
Secretory IgA (choice E) is the immunoglobulin associated with mucous membranes. Selective IgA
deficiency is the most common hereditary immunodeficiency. In this disorder, there is failure of the B cell
to switch the heavy chain class from IgM to IgA. Patients have an increased incidence of sinopulmonary
infections, diarrhea, allergies, and autoimmune diseases.
A 7-month-old infant is brought to your office because his mother believes that he has been “sick for
months.” The child appears to have bilateral otitis media as well as coarse rhonchi in his lungs, but
he is active and appears to be gaining weight. A complete blood count reveals normal numbers of T
2 ABDULRAHMAN BASHIR BENGHAZI CHILDREN HOSPITAL
28
25.1.2018
and B cells, but quantitative immunoglobulins demonstrate profoundly low levels of IgG, IgM, and
IgA. Your next step would be which of the following?
A. Refer the patient for bone marrow transplant.
B. Start scheduled intravenous immunoglobulin administration.
C. Initiate prophylactic antibiotic therapy.
D. Reassure the mother that the child should recover from immune function.
E. Anticipate that the child will develop chronic lung disease
Ans:-D
This patient has hypogammaglobulinemia, leading to chronic infection now that the placentally-acquired
immunoglobulin from his mother has decayed. As contrasted with X-linked agammaglobulinemia,
however, this patient has normal circulating numbers of B cells and the low levels of immunoglobulin are
due to a delay in onset of Ig production. This disorder is called transient hypogammaglobulinemia of
infancy. As depicted by the title, this disorder is not permanent; spontaneous recovery is expected by 2 to 4
years of age. Prophylactic antibiotics are not indicated, nor is prophylactic intravenous immunoglobulin
administration unless the patient continues to have serious recurrent infection.
In hypogammaglobulinaemia, which of the followings are true?
A. In nephrotic syndrome, the loss of immunoglobulin in urine is the only cause of predisposition to
infection.
B. In common variable immunodeficiency, there is an increased incidence of lymphoma.
C. Selective IgA deficiency is the commonest inherited form.
D. In patients over 40 years of age, a chest radiograph is mandatory to exclude Thymoma
E. The genetic defect in X-linked agammaglobulinaemia (XLA) is in Bruton’s tyrosine kinase (Btk)
Ans:- BCDE
In nephrotic syndrome, the susceptibility to pneumococcal infection is multifactorial and includes
decreased immunoglobulin levels, immunosuppressive treatment and in those patients with immune
complex mediated glomerulonephritis -a major factor is decreased levels of complement.
In XLA or Bruton’s agammaglobulinaemia, B-cell maturation is arrested at the pre-B-cell stage, suggesting
that Btk may be required to couple the intracellular signaling events mediated by the pre-B cell receptor.
Selective IgA deficiency is the commonest form of inherited Ig deficiency (Incidence about 1 in 800).
Although usually asymptomatic, the incidence is higher in patients with chronic lung disease suggesting
that IgA deficiency may be associated with increased respiratory infections. Selective IgA deficiency can
also occur after phenytoin and barbiturate administration as well as post toxoplamosis, measles or other
viral infections. Some patients with “selective” IgA deficiency may also have deficiencies in IgG2 and
IgG4. The commonest form of immunoglobulin deficiency is however, transient hypogammaglobulinaemia
of infancy. This presents between 6-12 months when the maternal antibody levels wane and the infant’s
own humoral system is not yet fully developed.
IgG subclass deficiency is associated with the following except:
A. IgG2 is associated with low IgA.
B. IgG2 is associated with poor antibody responses to polysaccharide antigens.
C. It is usually a permanent problem.
D. Severe IgG2 deficiency may lead to bronchiectasis.
E. Prophylactic antibiotics can be used to prevent severe infections
Ans:-C
This is a very rare condition that sometimes may be responsible for primary severe immune deficiency.
Children usually present with upper respiratory tract infection. It can be familial, and repeated infection
may cause bronchiectasis. It usually resolves spontaneously, and immunoglobulin is not needed except in
cases with severe recurrent infection. Prophylactic azithromycin or Augmentin is advisable. No routine
blood tests or chest X-rays are needed.
2 ABDULRAHMAN BASHIR BENGHAZI CHILDREN HOSPITAL
29
25.1.2018
You are evaluating a 1-year-old male in your office in post-hospitalization follow-up. He was
admitted with Pneumocystis jiroveci pneumonia 2 weeks ago. You also note that he has had several
respiratory infections with H. influenza and pneumococcus despite being immunized. Laboratory
evaluation notes a normal CBC. Flow cytometry demonstrates normal numbers of T and B
lymphocytes, but quantitative immunoglobulins show significantly decreased IgG and IgA levels,
although IgM levels are elevated. This immunodeficiency in most likely caused by which of the
following?
A. Maturation arrest of B cell precursors.
B. Abnormal function of CD4+ T helper cells.
C. Abnormal NK cell function.
D. Elevated levels of IgE.
E. Impaired class-switch recombination of immunoglobulins
Ans:-E
The elevated levels of IgM and profoundly low levels of IgG and IgA indicate a failure in immuneglobulin
class switching, making the patient susceptible to encapsulated organisms. The most common of these
disorders is X-linked hyper-IgM syndrome which causes an abnormality in the gene encoding the CD40
ligand on T cells leading to failure of CD40 on B cells leading to blockade of the class-switch process.
CD40 is also present on macrophages and dendritic cells, leading to susceptibility to opportunistic
organisms such as P. jiroveci.
X-linked hyper IgM syndrome :-
A. Causes defective B cell development
B. Is associated with normal levels of IgG
C. Can present with pneumocystis carinii infection
D. Is due to a defective CD40 ligand
E. Is due to a failure of Ig isotype switching
Ans:- CDE
The hyper-IgM syndrome is a rare, inherited immune deficiency disorder resulting from defects in CD40
ligand/CD40 interaction. X-linked hyper-IgM (type 1) is caused by defects in the CD40 ligand expressed
on activated T-cells. Autosomal recessive hyper-IgM (type 2) is caused by defects in the CD40-activated
RNA-editing enzyme- activation-induced cytidine deaminase. This enzyme is required for immunoglobulin
isotype switching and somatic hypermutation in B-cells.
In X-linked hyper-IgM, the loss of interaction between CD40 and its ligand results in impairment of T-cell,
B-cell and macrophage function. The interaction of CD40L on T-cells and CD40 on B-cells is essential for
B-cell activation and proliferation, isotype switching and formation of germinal centres (which are required
for affinity maturation of immunoglobulin). Therefore, although there are high levels of serum IgM which
can be produced without T-cell help, the T-dependent B-cell response is defective, and there is a lack of
IgG, IgA or IgE production. CD40L/CD40 interaction is also necessary for optimal activation of
macrophages by T-cells. The subsequent macrophage dysfunction contributes to the profound
immunodeficiency seen in these patients.
Patients are susceptible to infection with extracellular bacteria as well as intracellular pathogens. Boys
present with recurrent sinopulmonary infections and chronic diarrhea often caused by Giardia,
Campylobacter, rotavirus, or Cryptosporidium. These patients are particularly prone to opportunistic lung
infections caused by Pneumocystis carinii. There is an increased incidence of autoimmune diseases and
malignancies including adenocarcinomas of the liver and biliary tract.
Laboratory studies usually reveal normal B- and T-cell numbers, however neutropenia is common. IgM
levels are elevated but IgG and IgA levels are low. Diagnosis is confirmed by identifying the defective
gene. Treatment is supportive with IVIG and antibiotics.
A 14-year-old male with X-linked lymphoproliferative disease is admitted to the hospital with
fulminant Epstein–Barr viral infection, hepatitis, and respiratory failure. He is currently intubated,
2 ABDULRAHMAN BASHIR BENGHAZI CHILDREN HOSPITAL
30
25.1.2018
on ventilator and pressor support in the Pediatric Intensive Care Unit. He has received 2 g/kg of
intravenous immunoglobulin. Should he survive this current illness, you would expect which of the
following?
A. He is at great risk for extranodal lymphoid malignancy.
B. He will have a reactive, sustained polyclonal hypergammopathy.
C. He will develop chronic, active hepatitis.
D. His female offspring will be at great risk for fulminant EBV infection.
E. He will suffer significant neurologic impairment
Ans:-A
X-linked lymphoproliferastive disease is due to a mutation in SH2D1A, which encodes the protein SAP
(signaling lymphocyte activation molecule-associated protein), which regulates T cell response to EBV
infection. Patients can develop overwhelming EBV infection, which is fatal in over 50% of patients
infected. Survivors have sustained hypo- or agammaglobulinemia and can often develop extranodal
lymphoid malignancies, particularly of the Burkitt type. Other than as residua of severe infection, there are
no specifically associated neurocognitive abnormalities and patients do not develop chronic hepatitis
---------------------------------------------------------------------------------------------------------------------------------
Patients with IgA deficiency should be given which of the following with caution?
A. Packed RBCs.
B. Intravenous Immunoglobulin.
C. GM-CSF.
D. Pneumococcal immunization.
E. Folic acid supplementation
Ans:-B
IgA deficiency is the most common primary immunodeficiency, occurring in approximately 1 in 600
people. While most people with selective IgA deficiency are asymptomatic, some develop recurrent
infections at mucosal barriers, such as the respiratory and gastrointestinal tracts. Some patients with IgA
deficiency develop anti-IgA antibodies and are at risk for anaphylaxis when they receive any blood product
containing IgA, the most common of which is pooled intravenous immunoglobulin.
A 6-year-old male with juvenile idiopathic arthritis is brought to your office by his mother because
she is worried that his arthritis medicine “keeps making him sick.” He is currently taking naproxen,
calcium, and vitamin D. On review of his medical records, you note that he has had multiple
respiratory infections in the past several years, as well as a history of chronic otitis media requiring
tympanostomy tube placement which pre-dates his arthritis diagnosis. You suspect that he might
have an underlying immunodeficiency. Your advice to the mother would be which of the following?
A. The arthritis medication is worsening his immunodeficiency and will need to be stopped.
B. The arthritis and the immunodeficiency are unlikely to be related.
C. More aggressive treatment of his arthritis may increase the patient’s risk of infection.
D. Treatment of the arthritis will result in an improvement in his immunodeficiency.
E. The patient will require IgA supplementation.
Ans:-C
Given the chronic recurrent respiratory tract infections in an otherwise healthy child with arthritis, this
patient is likely to have IgA deficiency. The relationship of autoimmune disease and IgA deficiency has
been well described, particularly with juvenile idiopathic arthritis and celiac disease. There is no
treatment for IgA deficiency and treatment of the arthritis will not improve IgA production. Given the
destructive nature of untreated JIA, the medication cannot be stopped. More aggressive treatment for
arthritis, such as with methotrexate, steroids, or biologic agents will lead to further immunosuppression and
the patient will need to be monitored for infection and treated aggressively when present.
IgA deficiency:-
2 ABDULRAHMAN BASHIR BENGHAZI CHILDREN HOSPITAL
31
25.1.2018
A. Is an extremely rare finding.
B. May resolve spontaneously.
C. Can lead to increased respiratory & GIT infection
D. Is associated with autoimmune conditions.
E. Is commonly found in association with IgG3 subclass deficiency.
Ans:- BCD
In IgA deficiency, patients with more severe and recurrent sinopulmonary infection tend to have an
associated IgG2/IgG4 or IgG4 subclass deficiency.
Selective IgA deficiency:-
A. Is treated with iv sandoglobulin.
B. is very rare (incidence less than 1 in 10,000).
C. May present with autoimmune disease.
D. IgG2 subclass deficiency may also be present with more severe infective complications.
E. is associated with HLA-DR3.
Ans:-CDE
Selective IgA Deficiency is the commonest immune deficiency (affects 1 in 500-700 people). It is usually
sporadic, but can be inherited in an autosomal dominant or autosomal recessive pattern. There is an
association with MHC alleles HLA B8, DR3 and deletions of the C4A gene. Patients are often
asymptomatic. If present, clinical features are usually mild – recurrent sinusitis and respiratory infections
(mild) or chronic diarrhoea. Autoimmune syndromes (e.g. rheumatoid arthritis, Sjogrens disease, pernicious
anaemia, thyroiditis) are associated. Autoantibodies (organ-specific and non-specific) are often found even
in the absence of clinical disease. Anti-IgA antibodies occur in 30-40% and these may cause anaphylaxis if
transfusions are administered. Serum IgA levels are usually < 50µg/ml but serum IgG and IgM levels
usually normal. Some patients may have additional IgG subclass deficiencies. The defect in this syndrome
is a block in differentiation of IgA-expressing B cells to antibody- secreting plasma cells. Treatment is
symptomatic – antibiotics for infections.
IgA deficiency;-
A. Typically lead to sever Sinopulmonary infection.
B. Is associated with autoimmune disease.
C. Should be treated with intravenous immunoglobulin therapy.
D. Is feature of hereditary angio-oedema.
E. Is associated with atopy.
Ans; BE
Isolated IgA deficiency is the most common primary immunodeficiency. Most individual remain
asymptomatic, and those with recurrent infection tend to improve with age. Minor recurrent sinopulmonary
infection are the most common complication in childhood, however sever pneumonia and trouble ear
infection can occur. Infections at other sites are relatively rare and include increase the incidence of
Giardiasis. Allergic disease occur with an increased frequency in patient with immune deficiency
IgA deficiency:-
A. Is seen in Kartagener's syndrome
B. Is usually asymptomatic
C. May delay diagnosis of coeliac disease
D. Spontaneous recovery is a recognised feature
E. Affects both serum and secretory IgA levels
Ans:- ABCDE
IgA deficiency may be seen in:
i) CVID (all Ig groups)
ii) Bruton's agammaglobulinemia (all Ig groups)
2 ABDULRAHMAN BASHIR BENGHAZI CHILDREN HOSPITAL
32
25.1.2018
iii)Transient hypogammaglobulinemia of infancy (IgG,M+A)
iv) Ataxia telangiectasia (IgA+E)
v) Bloom's syndrome (IgA+M)
vi) Kartagener's syndrome (IgA)
vii) Selective IgA deficiency (IgA)
Selective IgA deficiency is common (1:350) but the incidence is not universal worldwide (rarer in Japan
1:1,800), affecting the levels of IgA (A1, A2 & secretory forms). Most people are completely
asymptomatic, but occasionally there is an increased incidence of respiratory, GI and GU infections.
Increasingly the diagnosis is been made following investigation of FTT where the diagnosis of coeliac
disease is made based on serology (antigliadin & endomyesial IgA) and therefore it is worth bearing in
mind in a negative coeliac screen where malabsorption seems likely and other conditions are similarly ruled
out. Alternatively, pre- and post- jejunal biopsy should be performed. In selective IgA deficiency there is an
increased incidence of cows' milk allergy, anaphylaxis, autoimmune disease and malignancy.
In transient hypogammaglobulinemia of infancy following the erosion of maternal passive immunity there
may be a delay of up to 36 months before full humoral immunity is restored. The levels of IgG & A are
predominantly affected, but also IgM.
Selective IgA deficiency associated with:-
A. Autosomal dominant
B. Associated with rheumatoid arthritis
C. Lymphoma is recognized complication
D. Autosomal recessive
Ans:-ABC
Selective IgA deficiency associated with:-
A. Ataxia telangectasia
B. Chronic mucocutaneous candidiasis
C. Rheumatoid arthritis
D. Phenytoin
Ans:- ACD
Selective IgA deficiency associated with:-
A. INH
B. Penicillamine
C. Tetracycline
D. Phenytoin
Ans:-BD
IgA deficiency is noted in patients treated with the same drugs associated with producing CVID (phenytoin,
d-penicillamine, gold, and sulfasalazine), suggesting that environmental factors may trigger this disease in a
genetically susceptible person.
IgA deficiency;
A. Is the commonest form of primary immunodeficiency
B. Usually present as growth faltering
C. Has an association with celiac disease
D. Is contraindication to BCG immunization
E. Increase the risk of transfusion reaction
Ans;-ACE
Selective IgA deficiency
A. Is inherited as an X- linked dominant
B. Occurs in about 1 in 400 000 people
C. Severs T cell deficiency almost always coexist
2 ABDULRAHMAN BASHIR BENGHAZI CHILDREN HOSPITAL
33
25.1.2018
D. Routine gammglobuline is the treatment of choice
E. Patient have an increased incidence of allergy, sinopulmonary infection gastrointestinal disease and
autoimmune disease
Ans:-E
IgA deficiency:
A. is the commonest form of primary immunodeficiency.
B. usually presents as growth faltering.
C. has an association with coeliac disease.
D. Has contraindication to BCG (bacille CalmetteGuerin) immunization.
E. increases the risk of transfusion reactions.
Ans:-ACE
A 17-month old girl presented with history of recurrent attacks of pneumonia since the age of 8
months. You suspect a predominant B-cell defect. Of the following, the BEST simple initial screening
test is
A. lgA measurement.
B. lgG measurement.
C. lgM measurement.
D. lgE measurement.
E. isohemoagglutinins titre
Ans:-A
If the lgA level is normal, selective lgA deficiency, which is the most common B cell defect, is excluded, as
are most of the permanent types of hypogammaglobulinemia, as lgA is usually very low or absent in those
conditions. if lgA is low, lgG and lgM should also be measured.
A 3 year-old girl presented with recurrent attacks of chest infection and frequent episodes of chronic
diarrhea, stool examination revealed Giardia lamblia and serum IgA level was very low. Regarding
this condition, all the following are trueEXCEPT
A. autosomal dominant inheritance.
B. phenotypically normal blood B cells.
C. administration of IVIG is not indicated.
D. environmental factors may trigger the disease.
E. it is associated with a celiac-like syndrome with dramatic response to gluten free diet
Ans:-E
An isolated absence or near absence (<10 mg/dl.) of serum and secretory lgA is the most common well-
defined immunodeficiency disorder. lgA deficiency is associated with a celiac-like syndrome, which may
or may not respond to a gluten-free diet.
---------------------------------------------------------------------------------------------------------------------------------
Which of the following is associated with common variable immunodeficiency?
A. Inflammatory bowel disease
B. Autoimmune disease
C. Normal levels of IgA
D. Granulomatous lesions
E. Susceptibility to recurrent parasitic infections
Ans:- ABD
CVID refers to a spectrum of disorders in which cell-mediated and antibody deficiency are combined in
variable amounts. Immunological findings are variable, but it is the humoral immune deficits that usually
present clinically. Some T-cell functional assays may be abnormal. Patients present with recurrent pyogenic
infections, and in some cases susceptibility to infections with Pneumocystis carinii, viruses or Candida.
Autoimmune disease occurs in about 10% of cases, usually immune thrombocytopenia, haemolytic
anaemia or SLE. There is an increased incidence of malignancy. Granulomas are a special feature of CVID
PEDIATRIC IMMUNOLGY MCQ
PEDIATRIC IMMUNOLGY MCQ
PEDIATRIC IMMUNOLGY MCQ
PEDIATRIC IMMUNOLGY MCQ
PEDIATRIC IMMUNOLGY MCQ
PEDIATRIC IMMUNOLGY MCQ
PEDIATRIC IMMUNOLGY MCQ
PEDIATRIC IMMUNOLGY MCQ
PEDIATRIC IMMUNOLGY MCQ
PEDIATRIC IMMUNOLGY MCQ
PEDIATRIC IMMUNOLGY MCQ
PEDIATRIC IMMUNOLGY MCQ
PEDIATRIC IMMUNOLGY MCQ
PEDIATRIC IMMUNOLGY MCQ
PEDIATRIC IMMUNOLGY MCQ
PEDIATRIC IMMUNOLGY MCQ
PEDIATRIC IMMUNOLGY MCQ
PEDIATRIC IMMUNOLGY MCQ
PEDIATRIC IMMUNOLGY MCQ
PEDIATRIC IMMUNOLGY MCQ
PEDIATRIC IMMUNOLGY MCQ
PEDIATRIC IMMUNOLGY MCQ
PEDIATRIC IMMUNOLGY MCQ
PEDIATRIC IMMUNOLGY MCQ
PEDIATRIC IMMUNOLGY MCQ
PEDIATRIC IMMUNOLGY MCQ
PEDIATRIC IMMUNOLGY MCQ
PEDIATRIC IMMUNOLGY MCQ
PEDIATRIC IMMUNOLGY MCQ
PEDIATRIC IMMUNOLGY MCQ
PEDIATRIC IMMUNOLGY MCQ
PEDIATRIC IMMUNOLGY MCQ
PEDIATRIC IMMUNOLGY MCQ
PEDIATRIC IMMUNOLGY MCQ
PEDIATRIC IMMUNOLGY MCQ
PEDIATRIC IMMUNOLGY MCQ
PEDIATRIC IMMUNOLGY MCQ
PEDIATRIC IMMUNOLGY MCQ

More Related Content

What's hot

Neonatologymcqs 100403011632-phpapp02
Neonatologymcqs 100403011632-phpapp02Neonatologymcqs 100403011632-phpapp02
Neonatologymcqs 100403011632-phpapp02AmarSETIA
 
Mcq contact diseseaes
Mcq contact diseseaesMcq contact diseseaes
Mcq contact diseseaesmonaaboserea
 
Mcq in neonatology
Mcq in neonatologyMcq in neonatology
Mcq in neonatologyVarsha Shah
 
Respiratory mcq rdmc_2018
Respiratory mcq rdmc_2018Respiratory mcq rdmc_2018
Respiratory mcq rdmc_2018Parthiv Mehta
 
Gastroenterology SCE MCQ
Gastroenterology SCE MCQGastroenterology SCE MCQ
Gastroenterology SCE MCQjuuraju
 
Practical pediatric quiz - Kaun Banega Winner
Practical pediatric quiz - Kaun Banega WinnerPractical pediatric quiz - Kaun Banega Winner
Practical pediatric quiz - Kaun Banega WinnerGaurav Gupta
 
Neonatology test
Neonatology testNeonatology test
Neonatology testVarsha Shah
 
Infectious diseases
Infectious diseasesInfectious diseases
Infectious diseasesAjay Agade
 
Gastrointestinal mcq
Gastrointestinal mcqGastrointestinal mcq
Gastrointestinal mcqRashed Hassen
 
MCQs - Urinary Tract Infection in Children
MCQs - Urinary Tract Infection in ChildrenMCQs - Urinary Tract Infection in Children
MCQs - Urinary Tract Infection in ChildrenDr Padmesh Vadakepat
 
Rwanda National council NCNM 2019 answer questions (1).pdf
Rwanda National council NCNM 2019 answer questions (1).pdfRwanda National council NCNM 2019 answer questions (1).pdf
Rwanda National council NCNM 2019 answer questions (1).pdfRBGroup
 
Mcq for neonatology
Mcq for neonatologyMcq for neonatology
Mcq for neonatologyVarsha Shah
 
Mcq. medical surgical nursing
Mcq. medical surgical nursingMcq. medical surgical nursing
Mcq. medical surgical nursingNursing Path
 
Internal Medicine Sample Questions
Internal Medicine Sample QuestionsInternal Medicine Sample Questions
Internal Medicine Sample QuestionsDJ CrissCross
 
Pulmonology mcqs -dr.ahmed_mowafy
Pulmonology mcqs -dr.ahmed_mowafyPulmonology mcqs -dr.ahmed_mowafy
Pulmonology mcqs -dr.ahmed_mowafyczer Shmary
 
50 mc qs on community health nursing part 3
50 mc qs on community health nursing part 350 mc qs on community health nursing part 3
50 mc qs on community health nursing part 3ManimegalaiSiva
 
Pathology Practice Examination
Pathology Practice ExaminationPathology Practice Examination
Pathology Practice ExaminationDJ CrissCross
 

What's hot (20)

Neonatologymcqs 100403011632-phpapp02
Neonatologymcqs 100403011632-phpapp02Neonatologymcqs 100403011632-phpapp02
Neonatologymcqs 100403011632-phpapp02
 
Mcq ped neuro
Mcq ped neuroMcq ped neuro
Mcq ped neuro
 
Mcq contact diseseaes
Mcq contact diseseaesMcq contact diseseaes
Mcq contact diseseaes
 
Mcq in neonatology
Mcq in neonatologyMcq in neonatology
Mcq in neonatology
 
Respiratory mcq rdmc_2018
Respiratory mcq rdmc_2018Respiratory mcq rdmc_2018
Respiratory mcq rdmc_2018
 
Gastroenterology SCE MCQ
Gastroenterology SCE MCQGastroenterology SCE MCQ
Gastroenterology SCE MCQ
 
Practical pediatric quiz - Kaun Banega Winner
Practical pediatric quiz - Kaun Banega WinnerPractical pediatric quiz - Kaun Banega Winner
Practical pediatric quiz - Kaun Banega Winner
 
Neonatology test
Neonatology testNeonatology test
Neonatology test
 
Infectious diseases
Infectious diseasesInfectious diseases
Infectious diseases
 
Gastrointestinal mcq
Gastrointestinal mcqGastrointestinal mcq
Gastrointestinal mcq
 
MCQs - Urinary Tract Infection in Children
MCQs - Urinary Tract Infection in ChildrenMCQs - Urinary Tract Infection in Children
MCQs - Urinary Tract Infection in Children
 
Rwanda National council NCNM 2019 answer questions (1).pdf
Rwanda National council NCNM 2019 answer questions (1).pdfRwanda National council NCNM 2019 answer questions (1).pdf
Rwanda National council NCNM 2019 answer questions (1).pdf
 
Mcq for neonatology
Mcq for neonatologyMcq for neonatology
Mcq for neonatology
 
Gastroenterology MCQs
Gastroenterology MCQsGastroenterology MCQs
Gastroenterology MCQs
 
Mcq. medical surgical nursing
Mcq. medical surgical nursingMcq. medical surgical nursing
Mcq. medical surgical nursing
 
Quiz gastro
Quiz   gastroQuiz   gastro
Quiz gastro
 
Internal Medicine Sample Questions
Internal Medicine Sample QuestionsInternal Medicine Sample Questions
Internal Medicine Sample Questions
 
Pulmonology mcqs -dr.ahmed_mowafy
Pulmonology mcqs -dr.ahmed_mowafyPulmonology mcqs -dr.ahmed_mowafy
Pulmonology mcqs -dr.ahmed_mowafy
 
50 mc qs on community health nursing part 3
50 mc qs on community health nursing part 350 mc qs on community health nursing part 3
50 mc qs on community health nursing part 3
 
Pathology Practice Examination
Pathology Practice ExaminationPathology Practice Examination
Pathology Practice Examination
 

Similar to PEDIATRIC IMMUNOLGY MCQ

05 -immunization_2
05  -immunization_205  -immunization_2
05 -immunization_2Sabat Tayfur
 
Immunization2
Immunization2Immunization2
Immunization2student
 
Human vaccinations in egypt 2021
Human vaccinations in egypt   2021Human vaccinations in egypt   2021
Human vaccinations in egypt 2021HusseinAbass1
 
Immunization special situations and AEFI
Immunization   special situations and AEFIImmunization   special situations and AEFI
Immunization special situations and AEFILokanath Reddy Mummadi
 
Human vaccinations ppt by dr. hussein abass
Human vaccinations  ppt by dr. hussein abassHuman vaccinations  ppt by dr. hussein abass
Human vaccinations ppt by dr. hussein abassHosin Abass
 
NEWER VIRAL VACCINES - presentation.pptx
NEWER VIRAL VACCINES - presentation.pptxNEWER VIRAL VACCINES - presentation.pptx
NEWER VIRAL VACCINES - presentation.pptxDr. Nagendra Kumar
 
WHO prequalified pentavalent vaccine
WHO prequalified pentavalent vaccineWHO prequalified pentavalent vaccine
WHO prequalified pentavalent vaccineAbhijeet Patil
 
Immumization in special situations
Immumization in special situationsImmumization in special situations
Immumization in special situationsOsama Arafa
 
Vaccinations schedule in Sri Lanka
Vaccinations schedule in Sri LankaVaccinations schedule in Sri Lanka
Vaccinations schedule in Sri LankaChamath Fernando
 
Hepatitis - Prevention and Management
Hepatitis - Prevention and ManagementHepatitis - Prevention and Management
Hepatitis - Prevention and ManagementArif Ismail
 
power point vaccination during pregnancy3faa.ppt
power point  vaccination during pregnancy3faa.pptpower point  vaccination during pregnancy3faa.ppt
power point vaccination during pregnancy3faa.pptIslamSaeed19
 
Immunization in children
Immunization in childrenImmunization in children
Immunization in childrengotolamy
 

Similar to PEDIATRIC IMMUNOLGY MCQ (20)

05 -immunization_2
05  -immunization_205  -immunization_2
05 -immunization_2
 
Immunization
ImmunizationImmunization
Immunization
 
Immunization2
Immunization2Immunization2
Immunization2
 
Human vaccinations in egypt 2021
Human vaccinations in egypt   2021Human vaccinations in egypt   2021
Human vaccinations in egypt 2021
 
Immunization special situations and AEFI
Immunization   special situations and AEFIImmunization   special situations and AEFI
Immunization special situations and AEFI
 
Human vaccinations ppt by dr. hussein abass
Human vaccinations  ppt by dr. hussein abassHuman vaccinations  ppt by dr. hussein abass
Human vaccinations ppt by dr. hussein abass
 
NEWER VIRAL VACCINES - presentation.pptx
NEWER VIRAL VACCINES - presentation.pptxNEWER VIRAL VACCINES - presentation.pptx
NEWER VIRAL VACCINES - presentation.pptx
 
WHO prequalified pentavalent vaccine
WHO prequalified pentavalent vaccineWHO prequalified pentavalent vaccine
WHO prequalified pentavalent vaccine
 
Immunization
ImmunizationImmunization
Immunization
 
Immumization in special situations
Immumization in special situationsImmumization in special situations
Immumization in special situations
 
Vaccinations schedule in Sri Lanka
Vaccinations schedule in Sri LankaVaccinations schedule in Sri Lanka
Vaccinations schedule in Sri Lanka
 
14 immunization(1)
14 immunization(1)14 immunization(1)
14 immunization(1)
 
Hepatitis - Prevention and Management
Hepatitis - Prevention and ManagementHepatitis - Prevention and Management
Hepatitis - Prevention and Management
 
Immunization-Program.pdf
Immunization-Program.pdfImmunization-Program.pdf
Immunization-Program.pdf
 
power point vaccination during pregnancy3faa.ppt
power point  vaccination during pregnancy3faa.pptpower point  vaccination during pregnancy3faa.ppt
power point vaccination during pregnancy3faa.ppt
 
Immunisation
ImmunisationImmunisation
Immunisation
 
Immunization
ImmunizationImmunization
Immunization
 
NIP-ppt.pptx
NIP-ppt.pptxNIP-ppt.pptx
NIP-ppt.pptx
 
Immunization in children
Immunization in childrenImmunization in children
Immunization in children
 
chapter 5 Immunization.pptx
chapter 5 Immunization.pptxchapter 5 Immunization.pptx
chapter 5 Immunization.pptx
 

Recently uploaded

Call Girl Coimbatore Prisha☎️ 8250192130 Independent Escort Service Coimbatore
Call Girl Coimbatore Prisha☎️  8250192130 Independent Escort Service CoimbatoreCall Girl Coimbatore Prisha☎️  8250192130 Independent Escort Service Coimbatore
Call Girl Coimbatore Prisha☎️ 8250192130 Independent Escort Service Coimbatorenarwatsonia7
 
Bangalore Call Girls Hebbal Kempapura Number 7001035870 Meetin With Bangalor...
Bangalore Call Girls Hebbal Kempapura Number 7001035870  Meetin With Bangalor...Bangalore Call Girls Hebbal Kempapura Number 7001035870  Meetin With Bangalor...
Bangalore Call Girls Hebbal Kempapura Number 7001035870 Meetin With Bangalor...narwatsonia7
 
Call Girls Service Surat Samaira ❤️🍑 8250192130 👄 Independent Escort Service ...
Call Girls Service Surat Samaira ❤️🍑 8250192130 👄 Independent Escort Service ...Call Girls Service Surat Samaira ❤️🍑 8250192130 👄 Independent Escort Service ...
Call Girls Service Surat Samaira ❤️🍑 8250192130 👄 Independent Escort Service ...CALL GIRLS
 
Kesar Bagh Call Girl Price 9548273370 , Lucknow Call Girls Service
Kesar Bagh Call Girl Price 9548273370 , Lucknow Call Girls ServiceKesar Bagh Call Girl Price 9548273370 , Lucknow Call Girls Service
Kesar Bagh Call Girl Price 9548273370 , Lucknow Call Girls Servicemakika9823
 
Bangalore Call Girls Nelamangala Number 7001035870 Meetin With Bangalore Esc...
Bangalore Call Girls Nelamangala Number 7001035870  Meetin With Bangalore Esc...Bangalore Call Girls Nelamangala Number 7001035870  Meetin With Bangalore Esc...
Bangalore Call Girls Nelamangala Number 7001035870 Meetin With Bangalore Esc...narwatsonia7
 
💎VVIP Kolkata Call Girls Parganas🩱7001035870🩱Independent Girl ( Ac Rooms Avai...
💎VVIP Kolkata Call Girls Parganas🩱7001035870🩱Independent Girl ( Ac Rooms Avai...💎VVIP Kolkata Call Girls Parganas🩱7001035870🩱Independent Girl ( Ac Rooms Avai...
💎VVIP Kolkata Call Girls Parganas🩱7001035870🩱Independent Girl ( Ac Rooms Avai...Taniya Sharma
 
Call Girls Bhubaneswar Just Call 9907093804 Top Class Call Girl Service Avail...
Call Girls Bhubaneswar Just Call 9907093804 Top Class Call Girl Service Avail...Call Girls Bhubaneswar Just Call 9907093804 Top Class Call Girl Service Avail...
Call Girls Bhubaneswar Just Call 9907093804 Top Class Call Girl Service Avail...Dipal Arora
 
Call Girl Number in Panvel Mumbai📲 9833363713 💞 Full Night Enjoy
Call Girl Number in Panvel Mumbai📲 9833363713 💞 Full Night EnjoyCall Girl Number in Panvel Mumbai📲 9833363713 💞 Full Night Enjoy
Call Girl Number in Panvel Mumbai📲 9833363713 💞 Full Night Enjoybabeytanya
 
VIP Call Girls Indore Kirti 💚😋 9256729539 🚀 Indore Escorts
VIP Call Girls Indore Kirti 💚😋  9256729539 🚀 Indore EscortsVIP Call Girls Indore Kirti 💚😋  9256729539 🚀 Indore Escorts
VIP Call Girls Indore Kirti 💚😋 9256729539 🚀 Indore Escortsaditipandeya
 
Top Rated Bangalore Call Girls Richmond Circle ⟟ 8250192130 ⟟ Call Me For Gen...
Top Rated Bangalore Call Girls Richmond Circle ⟟ 8250192130 ⟟ Call Me For Gen...Top Rated Bangalore Call Girls Richmond Circle ⟟ 8250192130 ⟟ Call Me For Gen...
Top Rated Bangalore Call Girls Richmond Circle ⟟ 8250192130 ⟟ Call Me For Gen...narwatsonia7
 
Book Paid Powai Call Girls Mumbai 𖠋 9930245274 𖠋Low Budget Full Independent H...
Book Paid Powai Call Girls Mumbai 𖠋 9930245274 𖠋Low Budget Full Independent H...Book Paid Powai Call Girls Mumbai 𖠋 9930245274 𖠋Low Budget Full Independent H...
Book Paid Powai Call Girls Mumbai 𖠋 9930245274 𖠋Low Budget Full Independent H...Call Girls in Nagpur High Profile
 
Best Rate (Hyderabad) Call Girls Jahanuma ⟟ 8250192130 ⟟ High Class Call Girl...
Best Rate (Hyderabad) Call Girls Jahanuma ⟟ 8250192130 ⟟ High Class Call Girl...Best Rate (Hyderabad) Call Girls Jahanuma ⟟ 8250192130 ⟟ High Class Call Girl...
Best Rate (Hyderabad) Call Girls Jahanuma ⟟ 8250192130 ⟟ High Class Call Girl...astropune
 
Call Girls Coimbatore Just Call 9907093804 Top Class Call Girl Service Available
Call Girls Coimbatore Just Call 9907093804 Top Class Call Girl Service AvailableCall Girls Coimbatore Just Call 9907093804 Top Class Call Girl Service Available
Call Girls Coimbatore Just Call 9907093804 Top Class Call Girl Service AvailableDipal Arora
 
(👑VVIP ISHAAN ) Russian Call Girls Service Navi Mumbai🖕9920874524🖕Independent...
(👑VVIP ISHAAN ) Russian Call Girls Service Navi Mumbai🖕9920874524🖕Independent...(👑VVIP ISHAAN ) Russian Call Girls Service Navi Mumbai🖕9920874524🖕Independent...
(👑VVIP ISHAAN ) Russian Call Girls Service Navi Mumbai🖕9920874524🖕Independent...Taniya Sharma
 
VIP Service Call Girls Sindhi Colony 📳 7877925207 For 18+ VIP Call Girl At Th...
VIP Service Call Girls Sindhi Colony 📳 7877925207 For 18+ VIP Call Girl At Th...VIP Service Call Girls Sindhi Colony 📳 7877925207 For 18+ VIP Call Girl At Th...
VIP Service Call Girls Sindhi Colony 📳 7877925207 For 18+ VIP Call Girl At Th...jageshsingh5554
 
VIP Russian Call Girls in Varanasi Samaira 8250192130 Independent Escort Serv...
VIP Russian Call Girls in Varanasi Samaira 8250192130 Independent Escort Serv...VIP Russian Call Girls in Varanasi Samaira 8250192130 Independent Escort Serv...
VIP Russian Call Girls in Varanasi Samaira 8250192130 Independent Escort Serv...Neha Kaur
 
(Rocky) Jaipur Call Girl - 9521753030 Escorts Service 50% Off with Cash ON De...
(Rocky) Jaipur Call Girl - 9521753030 Escorts Service 50% Off with Cash ON De...(Rocky) Jaipur Call Girl - 9521753030 Escorts Service 50% Off with Cash ON De...
(Rocky) Jaipur Call Girl - 9521753030 Escorts Service 50% Off with Cash ON De...indiancallgirl4rent
 
Russian Escorts Girls Nehru Place ZINATHI 🔝9711199012 ☪ 24/7 Call Girls Delhi
Russian Escorts Girls  Nehru Place ZINATHI 🔝9711199012 ☪ 24/7 Call Girls DelhiRussian Escorts Girls  Nehru Place ZINATHI 🔝9711199012 ☪ 24/7 Call Girls Delhi
Russian Escorts Girls Nehru Place ZINATHI 🔝9711199012 ☪ 24/7 Call Girls DelhiAlinaDevecerski
 
VIP Mumbai Call Girls Hiranandani Gardens Just Call 9920874524 with A/C Room ...
VIP Mumbai Call Girls Hiranandani Gardens Just Call 9920874524 with A/C Room ...VIP Mumbai Call Girls Hiranandani Gardens Just Call 9920874524 with A/C Room ...
VIP Mumbai Call Girls Hiranandani Gardens Just Call 9920874524 with A/C Room ...Garima Khatri
 

Recently uploaded (20)

Call Girl Coimbatore Prisha☎️ 8250192130 Independent Escort Service Coimbatore
Call Girl Coimbatore Prisha☎️  8250192130 Independent Escort Service CoimbatoreCall Girl Coimbatore Prisha☎️  8250192130 Independent Escort Service Coimbatore
Call Girl Coimbatore Prisha☎️ 8250192130 Independent Escort Service Coimbatore
 
Bangalore Call Girls Hebbal Kempapura Number 7001035870 Meetin With Bangalor...
Bangalore Call Girls Hebbal Kempapura Number 7001035870  Meetin With Bangalor...Bangalore Call Girls Hebbal Kempapura Number 7001035870  Meetin With Bangalor...
Bangalore Call Girls Hebbal Kempapura Number 7001035870 Meetin With Bangalor...
 
Call Girls Service Surat Samaira ❤️🍑 8250192130 👄 Independent Escort Service ...
Call Girls Service Surat Samaira ❤️🍑 8250192130 👄 Independent Escort Service ...Call Girls Service Surat Samaira ❤️🍑 8250192130 👄 Independent Escort Service ...
Call Girls Service Surat Samaira ❤️🍑 8250192130 👄 Independent Escort Service ...
 
Kesar Bagh Call Girl Price 9548273370 , Lucknow Call Girls Service
Kesar Bagh Call Girl Price 9548273370 , Lucknow Call Girls ServiceKesar Bagh Call Girl Price 9548273370 , Lucknow Call Girls Service
Kesar Bagh Call Girl Price 9548273370 , Lucknow Call Girls Service
 
Bangalore Call Girls Nelamangala Number 7001035870 Meetin With Bangalore Esc...
Bangalore Call Girls Nelamangala Number 7001035870  Meetin With Bangalore Esc...Bangalore Call Girls Nelamangala Number 7001035870  Meetin With Bangalore Esc...
Bangalore Call Girls Nelamangala Number 7001035870 Meetin With Bangalore Esc...
 
💎VVIP Kolkata Call Girls Parganas🩱7001035870🩱Independent Girl ( Ac Rooms Avai...
💎VVIP Kolkata Call Girls Parganas🩱7001035870🩱Independent Girl ( Ac Rooms Avai...💎VVIP Kolkata Call Girls Parganas🩱7001035870🩱Independent Girl ( Ac Rooms Avai...
💎VVIP Kolkata Call Girls Parganas🩱7001035870🩱Independent Girl ( Ac Rooms Avai...
 
Call Girls Bhubaneswar Just Call 9907093804 Top Class Call Girl Service Avail...
Call Girls Bhubaneswar Just Call 9907093804 Top Class Call Girl Service Avail...Call Girls Bhubaneswar Just Call 9907093804 Top Class Call Girl Service Avail...
Call Girls Bhubaneswar Just Call 9907093804 Top Class Call Girl Service Avail...
 
Call Girl Number in Panvel Mumbai📲 9833363713 💞 Full Night Enjoy
Call Girl Number in Panvel Mumbai📲 9833363713 💞 Full Night EnjoyCall Girl Number in Panvel Mumbai📲 9833363713 💞 Full Night Enjoy
Call Girl Number in Panvel Mumbai📲 9833363713 💞 Full Night Enjoy
 
VIP Call Girls Indore Kirti 💚😋 9256729539 🚀 Indore Escorts
VIP Call Girls Indore Kirti 💚😋  9256729539 🚀 Indore EscortsVIP Call Girls Indore Kirti 💚😋  9256729539 🚀 Indore Escorts
VIP Call Girls Indore Kirti 💚😋 9256729539 🚀 Indore Escorts
 
Top Rated Bangalore Call Girls Richmond Circle ⟟ 8250192130 ⟟ Call Me For Gen...
Top Rated Bangalore Call Girls Richmond Circle ⟟ 8250192130 ⟟ Call Me For Gen...Top Rated Bangalore Call Girls Richmond Circle ⟟ 8250192130 ⟟ Call Me For Gen...
Top Rated Bangalore Call Girls Richmond Circle ⟟ 8250192130 ⟟ Call Me For Gen...
 
Book Paid Powai Call Girls Mumbai 𖠋 9930245274 𖠋Low Budget Full Independent H...
Book Paid Powai Call Girls Mumbai 𖠋 9930245274 𖠋Low Budget Full Independent H...Book Paid Powai Call Girls Mumbai 𖠋 9930245274 𖠋Low Budget Full Independent H...
Book Paid Powai Call Girls Mumbai 𖠋 9930245274 𖠋Low Budget Full Independent H...
 
Russian Call Girls in Delhi Tanvi ➡️ 9711199012 💋📞 Independent Escort Service...
Russian Call Girls in Delhi Tanvi ➡️ 9711199012 💋📞 Independent Escort Service...Russian Call Girls in Delhi Tanvi ➡️ 9711199012 💋📞 Independent Escort Service...
Russian Call Girls in Delhi Tanvi ➡️ 9711199012 💋📞 Independent Escort Service...
 
Best Rate (Hyderabad) Call Girls Jahanuma ⟟ 8250192130 ⟟ High Class Call Girl...
Best Rate (Hyderabad) Call Girls Jahanuma ⟟ 8250192130 ⟟ High Class Call Girl...Best Rate (Hyderabad) Call Girls Jahanuma ⟟ 8250192130 ⟟ High Class Call Girl...
Best Rate (Hyderabad) Call Girls Jahanuma ⟟ 8250192130 ⟟ High Class Call Girl...
 
Call Girls Coimbatore Just Call 9907093804 Top Class Call Girl Service Available
Call Girls Coimbatore Just Call 9907093804 Top Class Call Girl Service AvailableCall Girls Coimbatore Just Call 9907093804 Top Class Call Girl Service Available
Call Girls Coimbatore Just Call 9907093804 Top Class Call Girl Service Available
 
(👑VVIP ISHAAN ) Russian Call Girls Service Navi Mumbai🖕9920874524🖕Independent...
(👑VVIP ISHAAN ) Russian Call Girls Service Navi Mumbai🖕9920874524🖕Independent...(👑VVIP ISHAAN ) Russian Call Girls Service Navi Mumbai🖕9920874524🖕Independent...
(👑VVIP ISHAAN ) Russian Call Girls Service Navi Mumbai🖕9920874524🖕Independent...
 
VIP Service Call Girls Sindhi Colony 📳 7877925207 For 18+ VIP Call Girl At Th...
VIP Service Call Girls Sindhi Colony 📳 7877925207 For 18+ VIP Call Girl At Th...VIP Service Call Girls Sindhi Colony 📳 7877925207 For 18+ VIP Call Girl At Th...
VIP Service Call Girls Sindhi Colony 📳 7877925207 For 18+ VIP Call Girl At Th...
 
VIP Russian Call Girls in Varanasi Samaira 8250192130 Independent Escort Serv...
VIP Russian Call Girls in Varanasi Samaira 8250192130 Independent Escort Serv...VIP Russian Call Girls in Varanasi Samaira 8250192130 Independent Escort Serv...
VIP Russian Call Girls in Varanasi Samaira 8250192130 Independent Escort Serv...
 
(Rocky) Jaipur Call Girl - 9521753030 Escorts Service 50% Off with Cash ON De...
(Rocky) Jaipur Call Girl - 9521753030 Escorts Service 50% Off with Cash ON De...(Rocky) Jaipur Call Girl - 9521753030 Escorts Service 50% Off with Cash ON De...
(Rocky) Jaipur Call Girl - 9521753030 Escorts Service 50% Off with Cash ON De...
 
Russian Escorts Girls Nehru Place ZINATHI 🔝9711199012 ☪ 24/7 Call Girls Delhi
Russian Escorts Girls  Nehru Place ZINATHI 🔝9711199012 ☪ 24/7 Call Girls DelhiRussian Escorts Girls  Nehru Place ZINATHI 🔝9711199012 ☪ 24/7 Call Girls Delhi
Russian Escorts Girls Nehru Place ZINATHI 🔝9711199012 ☪ 24/7 Call Girls Delhi
 
VIP Mumbai Call Girls Hiranandani Gardens Just Call 9920874524 with A/C Room ...
VIP Mumbai Call Girls Hiranandani Gardens Just Call 9920874524 with A/C Room ...VIP Mumbai Call Girls Hiranandani Gardens Just Call 9920874524 with A/C Room ...
VIP Mumbai Call Girls Hiranandani Gardens Just Call 9920874524 with A/C Room ...
 

PEDIATRIC IMMUNOLGY MCQ

  • 1. ABDULRAHMAN BASHIRE BEGHAZI CHILDREN HOSPITAL 11/12/2019 PEDIATRIC IMMUNIOLOGY MCQ
  • 2. 2 ABDULRAHMAN BASHIR BENGHAZI CHILDREN HOSPITAL 2 25.1.2018 ‫اھﺪاء‬ ‫ان‬ ‫ﷲ‬ ‫ﻣﻦ‬ ‫راﺟﯿﺎ‬ ‫اﻻطﻔﺎل‬ ‫اطﺒﺎء‬ ‫ﺟﻤﯿﻊ‬ ‫اﻟﻰ‬ ‫اﻟﻌﻤﻞ‬ ‫ھﺬا‬ ‫اﻗﺪم‬ ‫ﯾﻨﺎل‬‫ﺑﺎﻟﺮﺣﻤﺔ‬ ‫اﻟﺪﻋﺎء‬ ‫اﻻ‬ ‫وﻻاطﻠﺐ‬ ، ‫ورﺿﺎﺋﮭﻢ‬ ‫اﻋﺠﺎﺑﮭﻢ‬ ‫واﻟﻤﻐﻔﺮة‬‫روﺣﻰ‬ ‫وﺗﺆاﺋﻢ‬ ‫وﻋﻀﺪى‬ ‫ﻻﺧﻮى‬ "‫و‬ ‫اﺣﻤﺪ‬‫ﻋﺒﺪاﻟﻤﻨﻌﻢ‬" ‫ﻟ‬ ‫اﻏﻔﺮ‬ ‫اﻟﻠﮭﻢ‬‫ﮭﻢ‬‫ﺟﻨﺎﻧﻚ‬ ‫ﻓﺴﯿﺢ‬ ‫واﺳﻜﻨﮭﻢ‬ ‫وارﺣﻤﮭﻢ‬. ‫اﻣﯿﻦ‬
  • 3. 2 ABDULRAHMAN BASHIR BENGHAZI CHILDREN HOSPITAL 3 25.1.2018 Regarding HiB vaccination, which of the following are true? A. It is a live vaccine. B. It is contra-indicated in a child with a history of convulsions C. A booster is given with the MMR immunisation within one month of the first birthday. D. Vaccinations are given at 2, 3 and 4 months plus booster between 12 & 13 months. E. It must not be given as a single injection with DPT Ans:-CD DTaP/IPV/Hib is a combined vaccine which contains the following active ingredients: purified high dose diphtheria toxoid; purified tetanus toxoid; five purified components of the Bordetella pertussis bacteria; three strains of inactivated polio virus and a purified component of Haemophilus infiuenzae type b bacteria attached to a tetanus toxoid carrier protein. It is usually given to babies at the age of two, three and four months and again as a booster between 12 and 13 months. Since introduction of the HiB vaccine, the incidence of which of the following have conditions have decreased? A. Meningitis B. Otitis media C. Septic arthritis D. Epiglottitis E. Periorbital cellulitis Ans:-ACDE The HiB vaccine primarily prevents invasive HiB disease. In the past it had been a major cause of meningitis, septic arthritis, epiglottitis and peri-orbital cellulitis. It is logical therefore that the incidence of these diseases has declined since the introduction of the vaccine. Haemophilus influenzae type B is not recognised as a major cause of otitis media thus it is unlikely that the overall incidence of otitis media has been affected by the vaccine. The following are live vaccines A. BCG. B. Measles. C. Oral polio D. Pertussis. E. Rubella Ans:-ABCE Which vaccination is given by the intradermal route usually? A. Inactivated Polio B. Meningococcal C C. MMR. D. DTP. E. BCG Ans:-E BCG is the only vaccine given intradermally. The rest are given by the intramuscular or deep subcutaneous route Which of the following are true regarding vaccines? A. Diptheria vaccine is a toxoid.
  • 4. 2 ABDULRAHMAN BASHIR BENGHAZI CHILDREN HOSPITAL 4 25.1.2018 B. Pertussis vaccine is a killed bacillus. C. Injected polio vaccine is a live attenuated virus. D. BCG is a live attenuated bacillus. E. Measles is a live attenuated virus Ans:-ADE Previously virulent micro-organisms that have been inactivated (with chemicals or heat) include vaccines against influenza, cholera, plague, and hepatitis A. Most such vaccines may have incomplete or short-lived immune responses and are likely to require booster shots. Live, attenuated vaccines typically provoke more durable immunological responses. Examples include yellow fever, measles, rubella, and mumps. BCG is a live tuberculosis strain. The oral polio vaccine is a live attenuated virus but injected polio vaccine is inactivated. Inactivated toxic compounds (Toxoids) from micro-organisms include tetanus and diphtheria. The pertussis vaccine is acellular and made from part of the pertussis cell. Regarding immunisation in the UK:- A. The first dose of vaccines is given at a corrected age of 2 months in case of premature delivery B. Family history of febrile convulsion following immunisation is a relative contraindication C. Measles, mumps, rubella and polio are contraindicated in HIV +ve children D. Live vaccines should be avoided within 6 months of termination of lymphoma treatment E. Prolonged inconsolable crying to previous immunisation is a definite contraindication Ans:- DE Prematurity, asthma, chronic lung disease, congenital heart disease, and Downs syndrome are associated with an increased risk of complications from infectious diseases and they should be immunised as a matter of priority. Premature babies should be immunised according to the recommended schedule from 2 months after birth, irrespective of prematurely. Studies have shown that antibody response and adverse events are not significantly different in preterm and term infants immunised 2,3, and 4 months afterbirth. Prolonged inconsolable crying or high pitched screaming more than 4 hours is classified as a severe reaction to immunisation and is a contraindication to subsequent immunisation with same vaccine. Patients on active chemo or radiotherapy or within 6 months of termination of such treatment are contraindicated for live vaccines. They may not be able to produce a normal immune response to live vaccines and could potentially suffer from severe manifestation such as disseminated infection with BCG or paralytic polio from vaccine virus. No harmful effects have been reported with MMR and polio in HIV +ve individuals. However it should be noted that Polio is secreted for longer periods in HIV positive patients as compared to other people. Family history of febrile convulsion is not a contraindication to immunization A live vaccine shouldn’t be given to patient who :- A. Is finishing course of antibiotic therapy B. Received pooled immunoglobulin 2 month ago C. Has previous history of infection D. Is receiving high dose of corticosteroid E. Finished treatment for Hodgkin's disease 5 years ago Ans:-BD Live vaccine shouldn’t be given in the following circumstances:- Patient who have received chemotherapy or radiation within last year. Patient with malignant disorder such as Leukemia or lymphoma . Patient with immunosuppressive disorder Those receiving high dose of corticosteroid or immunoglobulin within the last 3 month It is appropriate to give live vaccine in the following;  Non-febrile common cold  Antibiotic treatment
  • 5. 2 ABDULRAHMAN BASHIR BENGHAZI CHILDREN HOSPITAL 5 25.1.2018  Previous history of the infection  Stable neurological condition If the patient had an acute illness the vaccination should be postponed until the fever has resolved The following are true regarding vaccine administration in children:- A. Live vaccines are contraindicted within 3 weeks of another live vaccine administration B. MMR can be given a week after administration of IV immunoglobulins C. Live vaccines are not given to children on prolonged steroid therapy D. Severe febrile illness is a contraindication for DPT E. Severe malnutrition is a contraindication for BCG Ans:-ACD Live vaccines are contraindicated 3 weeks before and 6 weeks after administration of Immunoglobulins. Other contraindications to live vaccines include –steroid administration / immunosuppressive therapy/ severe egg allergy/ immunodeficient conditions. Contraindications to pertussis vaccination include hypersensitivity to the vaccine and a severe reaction such as shock, persistent screaming, fever over 40.5 degree or serious neurological symptoms. Malnutrition, Stable neurological conditions, minor illness, chronic diseases of heart, lung and kidney, antibiotic treatment are not contraindications to vaccines. Which vaccination is NOT included in the 2, 3, 4 month ages of a child according to the UK immunisation schedule? A. Polio. B. Hib. C. DTP. D. MeningococcalC E. BCG Ans:-E BCG is usually given at birth in high risk neonates or at ages 10-14. Universal BCG immunisation of teenagers has recently been removed from the immunisation schedule, with a targeted strategy now recommended to immunise groups at increased risk in infancy, at birth or adulthood. Which of the following are true regarding BCG vaccination? A. Efficacy is demonstrated by a hypersensitivity response to vaccine administration. B. It is contraindicated in neonates. C. It offers protection against infection with Mycobacterium bovis as well as Mycobacterium TB. D. It should be injected subcutaneously. E. It contains a live attenuated virus Ans:-C Bacillus of Calmette and Guerin (BCG) is a vaccine against tuberculosis that is prepared from a strain of the attenuated (weakened) live bovine tuberculosis bacillus, Mycobacterium bovis (a bacterium, not a virus), that has lost its virulence in humans through serial propagation in artificial media. The bacilli retain sufficient immunogenicity to behave as a reasonably effective vaccine for the prevention of human tuberculosis, particularly in the prevention of TB meningitis and disseminated disease in early childhood. Protection against pulmonary disease in later life is less certain. BCG vaccine should always be given intradermally as subcutaneous administration is likely to be associated with adverse effects including local abscess formation and suppurative lymphadenitis. The Mantoux test results in a cutaneous hypersensitivity response to administered PPD (purified protein dericative), but neither a cutaneous response to the vaccine or to PPD are sufficient or required to demonstrate vaccine efficacy. With regard to Bacillus Calmette-Guerin (BCG): A. High-risk infants should have a positive Heaf test before immunisation proceeds.
  • 6. 2 ABDULRAHMAN BASHIR BENGHAZI CHILDREN HOSPITAL 6 25.1.2018 B. It is safe in asymptomatic HIV-positive patients. C. It may be given at the same time as other live vaccines. D. It should be given after a positive tuberculin test. E. Offers some protection against leprosy Ans:-CE High-risk infants do not require skin testing before immunisation up to the age of 3 months. BCG should be given only after a negative tuberculin test. It can be given at the same time as other live vaccines, otherwise a gap of 3 weeks should be observed. There is a risk that a suboptimal response to both may occur if this gap is not observed or the injections are not given at the same time. Live oral polio vaccine (OPV), which works by inducing gut immunity, is the exception and can be given at any time. HIV infection is an absolute contraindication to BCG vaccination. Which of the following children should be offered BCG immunisation? (Assume that any skin tests, where appropriate, have been done and proved negative) A. Normal healthy infants with Asian mothers going to Bangladesh. B. Migrant from Africa. C. A 13-year-old child who is moving to a high risk area but who has had immunisation as an infant. D. Child who has a contact with active pulmonary TB. E. Infant with a grandparent born in a high risk area. Ans:-ABDE BCG is recommended for the following groups if BCG immunisation, as evidenced by a characterstic scar, has not previously been carried out and they are negative for tuberculoprotein hypersensitivity: • all infants living in areas where the incidence of tuberculosis is greater than 40 per 100 000; • infants with a parent or grandparent born in a country with an incidence of tuberculosis greater than 40 per 100 000; • previously unvaccinated new immigrants from countries with a high incidence of tuberculosis; • contacts of those with active respiratory tuberculosis; • health service staff • veterinary staff • staff working in prisons, in residential homes and in hostels for refugees and the homeless; • those intending to stay for more than 1 month in countries with a high incidence of tuberculosis • neonates, infants, children or adults where immunisation is requested. A tuberculin skin test is necessary prior to BCG vaccination for: • all individuals aged six years or over • infants and children under six years of age with a history of residence or prolonged stay (more than three months) in a country with TB. • those who have had close contact with a person with known TB. • those who have a family history of TB within the last five years. Regarding tuberculin testing: A. It traditionally involves an injection into the extensor surface of the left forearm. B. The Heaf test should ideally be read between 48 and 72 hours. C. A positive result occurs when the area of induration is > 5 mm. D. Is negative if the Heaf grade is 1. E. Induration > 15 mm requires further investigation and possible antituberculous chemotherapy Ans:-CDE Tuberculin testing traditionally involves an injection into the flexor surface of the left forearm. The Heaf test is ideally read at 7 days (between 3 and 10 days) and the Mantoux test is read at 48-72 hours (but up to 96 hours). A positive result occurs when the area of induration is > 5 mm. N.B. the area of' flare' is irrelevant.
  • 7. 2 ABDULRAHMAN BASHIR BENGHAZI CHILDREN HOSPITAL 7 25.1.2018 The Heaf test is graded 0-4. Heaf grades 0-1 are negative and grades 2-4 are positive. Strongly positive reactions (i.e. Heaf grade 3-4 or induration > 15 mm) require further investigation and possible antituberculous chemotherapy. --------------------------------------------------------------------------------------------------------------------------------- MMR contraindications include which of the following? A. Child treated with 2mg/kg/day prednisolone for at least one week in the past month. B. Children with history of neonatal seizures. C. Previous vomiting after egg ingestion. D. History of febrile convulsions. E. Severe eczema Ans:-A Contraindications include: acute fever, recent immunoglobulin administration, administration of another live vaccine in the past 3 weeks, immunosuppression, allergy to neomycin or kanamycin, pregnancy. Steroids, such as prednisolone, given to children either orally or rectally at a daily dose of 2 mg/kg/day for at least one week or 1 mg/kg/day for one month should be deemed as a contraindication and administration on live vaccines should be postponed for at least 3 months after immunosuppressive treatment has stopped. In children with a significant history of an anaphylactic reaction to eggs, or who have had egg allergy and chronic severe asthma the vaccination should be administered in hospital. The vaccine is not however contraindicated in such patients. Which of the following is NOT a contra-indication to MMR vaccination: A. Children receiving high-dose cortico-steroids. B. Children with an anaphylactic allergy to egg. C. Children with an allergy to neomycin. D. Children with an acute febrile illness. E. Children who have received an immunoglobulin injection within the last 3 months Ans:-B All of the above are contra-indications as written in the BNF except egg allergy - this is a common misconception. In children with a significant history of an anaphylactic reaction to eggs, or who have had egg allergy and chronic severe asthma, the vaccination should be administered in hospital. The vaccine is not however contraindicated in such patients. Contraindications include: acute fever, recent immunoglobulin administration, administration of another live vaccine in the past 3 weeks, immunosuppression, allergy to neomycin or kanamycin, pregnancy. Steroids, such as prednisolone, given to children either orally or rectally at a daily dose of 2mg/kg/day for at least one week or 1 mg/kg/day for one month should be deemed as a contraindication and administration on live vaccines should be postponed for at least 3 months after immunosuppressive treatment has stopped. Which statements concerning the MMR vaccination are correct? A. Previous measles is a contraindication. B. The first dose is given at 6 months. C. A booster dose is given at 3-5 years. D. Parotid swelling is a recognised side effect E. Idiopathic thrombocytopenic purpura is a rare complication Ans:-CDE Any child who has had measles, mumps or rubella should be given the MMR vaccination regardless of previous infection (BNF). The first dose is given at 12-15 months with a pre-school booster. ITP is a rare complication and the risk of developing ITP is much less with MMR than with the actual diseases of measles, mumps and rubella. Parotid swelling is also a recognised side effect. With regard to the Measles, Mumps and Rubella (MMR) vaccine:
  • 8. 2 ABDULRAHMAN BASHIR BENGHAZI CHILDREN HOSPITAL 8 25.1.2018 A. It should not be given if there is a history of previous measles infection. B. It should not be given if there has been a previous measles immunization. C. It may be contraindicated in a child on oral corticosteroids. D. It is contraindicated where there is a personal history of febrile convulsions. E. It is contraindicated if there is a family history of epilepsy Ans:-C Live vaccine is contraindicated in children who may be immunosuppressed (such as oral steroids). Steroids, such as prednisolone, given to children either orally or rectally at a daily dose of 2mg/kg/day for at least one week or 1mg/kg/day for one month should be deemed as a contraindication and administration of live vaccines should be postponed for at least 3 months after immunosuppressive treatment has stopped. Previous infection or immunisation is not a contraindication, and 2 MMR doses are recommended. A fit within 72 hours of a previous dose would be a contraindication, although febrile convulsions and a family history of epilepsy are not. The measles-mumps-rubella (MMR) vaccine is: A. Contraindicated in children with egg allergy. B. A live vaccine. C. Contraindicated in children with autistic spectrum disorder. D. Recognised as having more side effects after the second dose. E. Contraindicated in a child with Di George syndrome Ans:-BE The MMR vaccine is a live vaccine and therefore is contraindicated in children with impaired cell-mediated immunity, as in Di George syndrome. Other contraindications for live vaccines include prednisolone (orally or rectally) at a daily dose of 2 mg/kg per day for at least 1 week or 1 mg/kg per day for a month, lower doses of steroid if used in conjunction with cytotoxic drugs, immunosuppression secondary to an underlying disease, immunoglobulin administration in the previous 3 months, children who have had a bone marrow transplant within 6 months and children being treated for malignant disease with chemotherapy /radiotherapy or who have completed treatment in the past 6 months. MMR is not contraindicated in egg allergy or in children with autistic spectrum disorder. Side effects include malaise, fever, rash within 7-10 days, parotid swelling 1%, febrile convulsion 1/1000. Arthropathy/ thrombocytopenia are rare. All side effects are less common after the second dose. Current immunisation information can be found. The MMR vaccine: A. Is contraindicated in patients allergic to neomycin. B. Should not be given within 3 weeks of another live vaccine (except oral polio vaccine). C. Is safe in pregnancy. D. Commonly results in a rash with or without fever from day 5 to day 10, lasting approximately 2 days. E. Is contraindicated in patients who have received an injection of immunoglobulin within 3 months Ans:-ABDE The MMR vaccine is contraindicated in patients who are allergic to neomycin or kanamycin. In children who have had a previous anaphylactic reaction to egg, immunisation is not absolutely contraindicated and should be discussed with a local paediatrician or immunisation coordinator. The MMR vaccine should not be given within 3 weeks of another live vaccine, because this results in a suboptimal response. Likewise, it is contraindicated in patients who have received an injection of immunoglobulin within 3 months, because no response will be mounted in the presence of immunoglobulin that may contain antibodies to measles, mumps or rubella. Pregnancy should be avoided for at least 1 month after immunisation, which may well result in a rash with or without fever from about day 5-10 lasting about 2 days. It is therefore sensible to provide advice on temperature control at the time of immunisation. --------------------------------------------------------------------------------------------------------------------------------
  • 9. 2 ABDULRAHMAN BASHIR BENGHAZI CHILDREN HOSPITAL 9 25.1.2018 Which of the following is correct regarding human varicella zoster immunoglobulin (VZIG)? A. Is used to treat severe chicken pox infection B. Is recommended for all patients with eczema exposed to chickenpox. C. Is invariably protective against severe varicella. D. Should be given to a 6 week old baby whose mother has developed chickenpox E. Should be given to an 18 week pregnant non-immune female who has been exposed to a case of chicken pox. Ans:-E Varicella has a secondary infection rate in household contacts of 90%. It is commonest in spring time, and the incubation period is 14-21 days. It shares the herpes virus family properties of latency and reactivation (zoster). Risks to the fetus and neonate relate to the time of infection:  Less than 20 weeks pregnancy: congenital varicella (limb hypoplasia, microcephaly, cataracts, growth retardation, skin scarring). High mortality.  Second to third trimester: herpes zoster in an otherwise healthy infant.  Minus 7 days to plus 7 days after delivery: severe and even fatal disease (30% mortality). Although a live attenuated vaccine is available, it is not licensed for use in the UK. Varicella zoster immunoglobulin is prepared from pooled plasma of UK blood donors with a history of recent chickenpox or herpes zoster. Being an immunoglobulin, it is a protein concentrate, and should be stored between 2 and 8oC. Donors are screened for HIV, hepatitis B and hepatitis C. VZIG prophylaxis is recommended for patients who fulfil all the following criteria:  A clinical condition that increases the risk of severe varicella, (e.g. immunosuppression, neonates, pregnant women).  No antibodies to varicella zoster.  Significant exposure to chickenpox or herpes. Severe or fatal varicella can occur despite VZIG prophylaxis. Active immunisation should therefore be used for susceptible immunosuppressed patients at long term risk. Clinical chickenpox occurs in 50% of those who receive VZIG prophylaxis, and 10% more will be affected sub-clinically. Which of the following are true regarding Oral Polio Vaccine (OPV)? A. It contains 3 strains of live attenuated virus. B. It is considered ineffective in the presence of diarrhea. C. It is contraindicated in children on oral steroids 2mg/kg/day D. Use of the inactive vaccine reduces the carriage of the wild virus. E. Incidence of vaccine associated poliomyelitis is negligible Ans:-ABC Oral polio ('Sabin') vaccine is given by mouth in contrast to the inactivated ('Salk') vaccine (IPV) which is given IM. Polio vaccines are usually given at two months, three months and four months of age, with a booster before school usually between 3 and 5 years of age and again, before leaving school, between 15 and 19 years old. Boosters thereafter are not normally necessary, unless travelling to an area where polio is common, or likely to be exposed to people with polio. The oral vaccine contains live virus particles which have been attenuated to reduce the risk of neurological disease. The risk of vaccine associated poliomyelitis is small but not negligible and this together with the elimination of wild polio from the European Region was the principle reason for the change to the routine use of IPV in the UK in 2006. Three types of poliomyelitis virus (Types 1, 2 and 3) are included in the vaccine. Precaution:- Postpone if acute illness with pyrexia/diarrhoea/vomiting, Immunodeficiency/treatment with high doses of steroids/immunosuppresants, First four months of pregnancy
  • 10. 2 ABDULRAHMAN BASHIR BENGHAZI CHILDREN HOSPITAL 10 25.1.2018 Which of the following are true regarding the Meningococcal vaccine which is used in the UK immunisation schedule? A. It protects against types B and C. B. It can cause meningoencephalitis. C. It is contraindicated in HIV infection. D. 3 injections are given from 6 months of age. E. It is a live attenuated vaccine Ans:-D Almost all childhood meningococcal disease in the UK is caused by Neisseria meningitidis serogroups B and C. Meningococcal Group C conjugate vaccine protects only against infection by serogroup C; it can be given from 2 months of age. After early adulthood the risk of meningococcal disease declines, and immunisation is not generally recommended after the age of 25 years. Meningococcal Group C conjugate vaccine provides long-term protection against infection. The recommended schedule consists of 3 doses given at 3 months, 12 months and as a booster at around 14 years of age. Side-effects of meningococcal Group C conjugate vaccine include redness, swelling, and pain at the site of the injection, mild fever, irritability, drowsiness, dizziness, nausea, vomiting, diarrhoea, headache, myalgia, rash, urticaria, pruritus, malaise, lymphadenopathy, hypotonia, paraesthesia, hypoaesthesia, and syncope. Hypersensitivity reactions and seizures have been reported rarely. Symptoms of meningism have also been reported rarely, but there is no evidence that the vaccine causes meningococcal C meningitis. There have been very rare reports of Stevens-Johnson syndrome. The CSM has advised that vaccination provides benefit in terms of lives saved and disabilities prevented. Currently meningococcal vaccine is a purified heat stable extract from the polysaccaride outer capsule of Neisseria meningitidis. --------------------------------------------------------------------------------------------------------------------------------- Which of the following are absolute contra-indications to Pertussis immunisation? A. Epilepsy. B. Family history of convulsions after immunization. C. Eczema. D. Autism. E. Previous severe local reaction to immunisation Ans:-All false Since 2006 acellular pertussis vaccines have been used for routine immunisation in the UK and this change, together with an assessment of vaccine reactions has led to a significant change in the contraindications and warnings relating to pertussis immunisation. The only remaining absolute contraindications for pertussis-containing vaccines are: • Confirmed anaphylactic reaction to a previous dose of pertussis-containing vaccine, or • Confirmed anaphylactic reaction to neomycin, streptomycin or polymyxin B (which may be present in trace amounts in the vaccine). Of particular note (and in contrast to previous advice), immunisation with pertussis-containing vaccines should not be withheld despite a previous history of convulsions, high fever, hypotonic-hyporesponsive episodes (HHE), persistent crying, screaming or severe local reaction following immunisation. By the time of school entry children in the UK should have received 4 doses of pertussis-containing vaccines (DTaP/Hib/IPV at ages 2, 3 & 4 months, and DTaP or dTaP at age 3.5 to 4 years (a preschool booster). The booster dose at school entry was added in 2006 at the same time that the change was made from whole-cell to acellular pertussis component vaccines. A boy was born at 26 weeks gestation. He had severe hyaline membrane disease and was ventilated for 3 weeks. He is now three months old and has just been discharged from hospital. He should receive pertussis vaccination even if:
  • 11. 2 ABDULRAHMAN BASHIR BENGHAZI CHILDREN HOSPITAL 11 25.1.2018 A. His Apgar scores were 3 at 1, and 5 at 5 minutes. B. He had a pneumothorax and 4 convulsions in the first week. C. He was hypoglycaemic and very jittery for a short period. D. After a pneumothorax, a cranial ultrasound scan showed an intraventricular haemorrhage with some loss of cerebral substance. E. He continues to have uncontrolled convulsions two or three times a week Ans:-ABCD Children with cerebral damage, a personal history of convulsions, or a family history of febrile convulsions are at increased risk of a febrile fit following pertussis and measles immunisation. However, these are not absolute contraindications and they are not at any greater risk of permanent adverse effects from the vaccines and should receive them. Children whose epilepsy is poorly controlled should not be immunised until control is achieved, although convulsions themselves are not a contraindication. The only absolute contraindications for pertussis-containing vaccines are: • Confirmed anaphylactic reaction to a previous dose of pertussis-containing vaccine, or • Confirmed anaphylactic reaction to neomycin, streptomycin or polymyxin B (which may be present in trace amounts in the vaccine). Contraindication to pertussis immunisation include:- A. Acute febrile illness B. Family history of seizures C. Known cerebral palsy D. Generalized reaction to previous dose E. Autism Ans:-AD A two-month-old child had his first DTP injection 3 days previously and his mother has returned because his thigh is swollen. On examination, most of the anterolateral surface of the thigh is red and indurated. What will you recommend for this child in the future? A. He should not have any further immunizations B. He should have all immunisations except pertussis C. He should receive the acellular pertussis vaccine when next immunized D. He should receive the oral pertussis vaccine when next immunized E. He should receive the whole cell pertussis vaccine (as DTP) as usual at his next immunization Ans:- C This degree of thigh swelling constitutes a severe local reaction (official definition: 'an extensive area of redness which becomes indurated and involves most of the anterolateral surface of the thigh or a major part of the circumference of the upper arm'). In the case of a severe local reaction, the acellular pertussis vaccine should be administered the next time. This is composed of a selection of proteins from pertussis and is said to have fewer side effects than the whole cell pertussis vaccine (which is composed of whole killed organism). A severe generalised reaction is defined as any of the following: * A temperature >39.5C within 48 hours of pertussis vaccination * Seizures or encephalopathy within 72 hours of pertussis vaccination * Prolonged unresponsiveness or collapse * Anaphylaxis or bronchospasm * Inconsolable screaming for more than 4 hours within 2 days of immunization. In the case of a severe generalised reaction, diphtheria and tetanus ONLY should be given next time. There is no oral pertussis vaccine. Any oral vaccine will be a live vaccine (e.g. oral typhoid, oral polio (Sabin)). Childhood immunisation with pertussis is absolutely contraindicated in:
  • 12. 2 ABDULRAHMAN BASHIR BENGHAZI CHILDREN HOSPITAL 12 25.1.2018 A. Immunocompromised patients. B. Down Syndrome. C. Cerebral palsy. D. Family history of convulsion. E. Rectal temperature of 38.5°C. Ans:- all false Comments: Killed vaccine Contraindications: Relative: acute illness. Severe local - use acellular vaccine. Absolute: Severe generalised reaction (fever >39.5 within 48 hours), bronchospasm,laryngeal oedema,collapse, prolonged unresponsiveness, prolonged inconsolable crying for more than 4hours convulsions/encephalopathy within 72 hours. Which of the following children should not receive live vaccines? A. Those requiring regular intravenous Immunoglobulin therapy. B. Those on a prednisolone dose of 3 mg/kg per day for longer than 1 week. C. Children vvith graft-versus-host disease. D. Children on chemotherapy. E. Children who have had a solid organ transplant. Ans:-ABCDE --------------------------------------------------------------------------------------------------------------------------------- The following are notifiable diseases: A. Acquired immune deficiency syndrome (AIDS). B. Mumps. C. Tuberculosis (TB). D. Rubella. E. Malaria Ans:-BCDE Which of the following are indications for influenza vaccination in children? A. Spastic diplegia. B. AIDS. C. Acute lymphoblastic leukaemia. D. Splenectomy. E. Bronchiectasis. Ans:-BCDE The injectable influenza vaccination is recommended for all children > 6 months old in high risk groups (chronic lung disease, chronic heart disease, chronic renal disease, immunosuppression (e.g such that occurs in leukaemia, AIDS and splenectomy) and diabetes). An annual nasal spray influenza vaccine was introduced into the NHS schedule in 2013 for 2 and 3 year olds. The programme will eventually be extended to all children between 2 and 16 years of age in the UK. It is safe for HIV-infected children to have the following immunisations: A. Measles-mumps-rubella (MMR). B. Bacille Calmette-Guerin (BCG) . C. Haemophilus influenzae type B (HIB) D. Diphtheria-pertussis-tetanus (DPT) E. Hepatitis B Ans:-ACDE
  • 13. 2 ABDULRAHMAN BASHIR BENGHAZI CHILDREN HOSPITAL 13 25.1.2018 Current recommendations are that it is safe for HIV-infected children to receive: MMR; oral polio (inactivated form may be given), pertussis, diphtheria, tetanus, typhoid, cholera, hepatitis B and HIB. They should not have BCG, yellow fever and oral typhoid. It is advised by department of Health that children who are HIV positive and who have symptoms of AIDS can receive the following vaccine:- A. MMR B. Hib C. Pertussis D. Oral typhoid E. BCG Ans:- ABC Live vaccine:-MMR, Poliomyelitis Inactivated vaccine:-cholera, Diphtheria, Hib. Hepatitis A &B, Influenza, Meningococcal, Pertussis, Pneumococcal, Poliomyelitis, Tetanus, Typhoid ( injection), BCG vaccine, yellow fever, oral typhoid are C/I in symptomatic HIV patient. A fully immunized 5-year-old has a splenectomy for spherocytosis. Which of the following vaccines are indicated? A. Hepatitis B. B. BCG. C. Meningococcal group C vaccine. D. Pneumococcal. E. Influenza. Ans:-DE Vaccinations against pneumococcus should be given before a splenectomy, and yearly vaccinations against influenza are recommended after a splenectomy. ----------------------------------------------------------------------------------------------------------------------------- Members of the innate immune system include: A. T Iymphocytes. B. antimicrobial peptides. C. Insulin-like growth factors. D. Complement C3. E. Lectin. Ans:-BD The innate or non-specific immune system is the first line of defence against pathogen. It has many components, including mechdnical barriers (skin, mucou secretions) and soluble factors (complement, mdnnose-binding lectin) as well as neutrophils and macrophages. Anti-microbial peptides, such as beta defensins are proteins secreted by endothelial cells. Specific immunity are provided by T cells and B cells, and results in the generation of immune memory, resulting in rapid responses on subsequent challenge by a given pathogen. T lymphocytes:- A. Are derived from a precursor cell in the bone marrow . B. 95% die in the thymus. C. All have the alpha-beta heterodimeric T cell receptor. D. Are usually activated by intact protein antigens . E. Are prominent in type II hypersensitivity reactions Ans:- AB
  • 14. 2 ABDULRAHMAN BASHIR BENGHAZI CHILDREN HOSPITAL 14 25.1.2018 T lymphocytes comprise 70-80% of the lymphocyte population and are haematopoietic cells. They are produced in bone marrow and paracortical regions of lymph nodes and spleen. Maturation occurs in the thymus, and involves a complicated series of positive and negative selection steps coordinated by the thymocytes and dendritic cells within the thymus, resulting in apoptosis of unwanted cells. The TCR is a heterodimer of two chains (alpha and beta, or gamma and delta) linked by disulphide bonds and is found at the cell surface associated with a complex of other polypeptides known collectively as CD3. The two types of TCR are called TCR-1 (gamma-delta) and TCR-2 (alpha-beta). TCR-1 cells are thought to have a restricted repertoire and to be mainly non-MHC restricted. TCR-2 cells recognise peptide fragments associated with MHC molecules. The following are known functions of T helper cells :- A. Production of T cell growth factors such as IL-2 B. Promotion of immunoglobulin isotype class switching C. Production of perforin D. Production of suppressive cytokines such as IL-10, TGF-beta E. Induction of the maturation of immature dendritic cells Ans:-ABDE Perforin is produced by cytotoxic T lymphocytes (CTL) to kill target cells. Helper T lymphocytes (HTL) are usually CD4+ T-cells. HTL can be sub-classified into TH1 and TH2 cells. More recently, further sub-populations of CD4+ T cells have been identified, termed Th3 and Tr1 cells. In general, TH-1 cells secrete pro-inflammatory cytokines such as IL-2, IFN-gamma, TNF-alpha , TNF-beta and are important for cell mediated immunity. They also promote the production of opsonising antibodies. TH-2 cells secrete IL-4, IL-5 and IL-10 and are important for humoral responses. TH-3 and Tr1 are not yet well defined but the former secrete predominantly IL-10 while the latter secrete mainly TGF-beta. It must be remembered that this differential cytokine expression is not absolute. Which type of T-cells have the major role of killing host cells infected by pathogens A. TH-1 (CD4). B. TH-2 (CD4). C. TC (CD8). D. TH-1 (CD8). E. TH-2 (CD8) Ans: C T cells are divided into three functional classes:  TC (CD8) cells kill host cells infected by pathogens—notably viruses—that replicate within the cytoplasm of host cells;  TH-1 (CD4) cells activate macrophages and thus allow them to destroy pathogens such as M tuberculosis and P carinii, which inhabit macrophage vesicles;  TH-2 (CD4) cells activate B cells to produce antibodies . Thus, T lymphocytes are critical in both humoral and cell-mediated immunity, and deficiencies can be devastating. Which type of T-cells have the major role of activating macrophages thus allowing them to destroy pathogens such as M tuberculosis and P carinii A. TH1 (CD4) B. TH2 (CD4) C. TC (CD8) D. TH1 (CD8) E. TH2 (CD8) Ans:- A The following contribute to the natural immunity transmitted in breast milk: A. Macrophages
  • 15. 2 ABDULRAHMAN BASHIR BENGHAZI CHILDREN HOSPITAL 15 25.1.2018 B. natural killer cells. C. Lysozyme. D. mast cells. E. secretory IgA. Ans:-ABCE The following HLA associations are correct: A. HLA DR3 and dermatitis herpetiformis. B. HLA A3 and schizophrenia. C. HLA DR2 and Goodpasture’s syndrome. D. HLA DQW2 and coeliac disease. E. HLA DR7 and Hashimoto’s thyroiditis Ans:- ACD Certain HLA alleles are associated with increased risk of disease. The following are further examples: A3/B14 in haemochromatosis; A28 in schizophrenia: B5 in Behçet’s disease and ulcerative colitis; Dfl3,7 DQW2 in coeliac disease; B27 in ankylosing spondylitis and psoriatic arthropathy: DR2 in Goodpasture’s syndrome and narcolepsy; DR3 in chronic active hepatitis, Grave’s disease, myasthenia gravis, Addison’s disease and Sjôgren’s syndrome: DR4 in rheumatoid arthritis and insulin dependent diabetes, and DR5 in Hashimoto’s thyroiditis. The following statements are true regarding superantigens: A. They are very strongly antigenic bacterial polysaccharides. B. They bind to B cell receptors. C. Lead to excess stimulation of T cell proliferation and interleukin-2 production. D. They are responsible for gram-negative toxic shock. E. Lead to stimulation of production of TNFa and other cytokines. Ans:-CE Superantigens are an unusual type of bacterial toxin comprising of proteins which exert their effect by forming a bridge between the MHC complex of antigen-presenting cells and T cell receptors. Because the binding to T cell receptors is indiscriminate, excess stimulation of T cell proliferation occurs leading to abnormally high levels of IL-2,with symptoms of nausea, vomiting, fever and malaise. They have a role in Gram-positive septic shock, particularly toxic shock syndrome and staphylococcal food-borne disease. It is possible that they may also have a role in the pathogenesis and clinical features of Kawasaki disease. Which of the following regarding the Fc component of antibody are true? A. Determines antigen bindinq capecificity. B. Determines the ability to cross the placenta. C. Determines the ability to bind with mast cells. D. When produced in excess, excreted in urine as Bence-Jones protein. E. Determines metabolic half life -of the whole molecule Ans:-BCE The Fc component of antibody composed of two heavy chain and it bind to various cell receptors and complement proteins. It mediate different physiological effects of antibodies ( opsonization, cell lysis, mast cell, basophil and eosinophils degranulation and other process. Bence –Jones protein are free immunoglobulin light chains. The secondary immunoglobulin response is not characterized by:- A. Isotype switching to IgG B. Production of immunoglobulin with higher affinity to the antigen C. Production of memory plasma cells D. A quantitatively increased antibody production as compared to the primary immune response E. Requirement of help from T-cells
  • 16. 2 ABDULRAHMAN BASHIR BENGHAZI CHILDREN HOSPITAL 16 25.1.2018 Ans:- C Memory B-cells are responsible for the secondary Ig response. Memory B-cells differ qualitatively as well as quantitatively from naïve B cells. After priming during the primary immune response, the frequency of antigen-specific B-cells may increase 10-100 fold. Therefore the secondary immune response to a specific antigen occurs more rapidly than the primary response, and larger amounts of antibody are produced. Memory B-cells also have higher affinity for antigen as they have undergone affinity maturation in the germinal centres of lymphoid tissue. Memory B- lymphocytes have undergone class switch and produce mainly IgG but also other immunoglobulin isotypes. Both primary and secondary immunoglobulin responses require T-cell help. (NB. Certain bacterial antigens can activate B cells directly, e.g. LPS). Plasma cells are the effector cells of both primary and secondary immune responses. They secrete large amounts of antibody, but are short lived cells in order to limit antibody responses. IgA:- A. is found in breast milk. B. is important in mucosal immunity. C. can fix complement. D. is the most abundant immunoglobulin in the blood. E. is deficit in coeliac disease. Ans:-ABE IgA: A. Is present in breast milk and saliva. B. Isolated deficiency is very serious. C. Can fix complement. D. Is the most abundant in blood. E. Has two subclasses and secretory component. Ans:-AE The following immunoglobulin are responsible for: A. IgA leads to bacterial agglutination. B. IgD is involved in inhibition of viral replications. C. IgE : Type 1 sensitivity reaction D. IgG : neutralisation of bacterial toxins E. IgM : complement activation. Ans:-ACDE The following statements are correct: A. IgA can activate complement via the alternative pathway. B. IgA normally represents less than 1% of the serum immunoglobulin pool. C. J-chains are associated with both lgA and gM. D. the Fab fragment determines the class of immunogiobulin. E. IgE can cross the placenta. Ans:- AC Secretory IgA is dimeric. the two subunits joined by a J-chain; IgM is pentameric. IgA and IgE can fix complement via the alternative pathway: IgG and IgM via the classical pathway. Cf the normal total immuncglobulin pool IgG makes up approx 75%, lgA 15% arid gM 10%. There are traces of IgD and IgE. Only lgG can cross the placenta. The constant region of the molecule (Fc) determines the class of immunoglobulin. The Fab region has a variable (V) and constant (OH) domain, the former being unique for each antibody. Immunoglobuline G ; A. Has MW of 900 000 dalton B. Is the first immunoglobuline class to raise in acute infection
  • 17. 2 ABDULRAHMAN BASHIR BENGHAZI CHILDREN HOSPITAL 17 25.1.2018 C. Has half life of 21 days. D. Fall in the first few weeks of life. E. Is the main immunoglobuline class involved in the ABO blood group system Ans;CD Which of the following statement regarding immunoglobulin classes and function are Correct A. IgM is pentamer. B. IgA is always dimmer. C. IgD is T cell receptor. D. IgG4 is responsible for polysaccharide response. E. All IgG classes are involved in complement fixation pathway Ans ; A Passage of IgG through the placenta is determined by:- A. Molecular weight. B. Fc fragment. C. JA chain. D. Secretary complement Ans:- B Which statement about immunoglobulin is false A. IgG comprise 70-80% of the total. B. IgG & IgM can both cross the placenta in human. C. The half life of IgG is about 25 days. D. The spleen seems to play an important role in producing IgM antibody. E. IgA comprise 10-15% of the total and is found mainly in the secretion Ans ;B C reactive protein ; A. Is synthesized in the liver. B. Production is stimulated by interleukin-6. C. Freely crosses the placenta. D. Concentration raises following intraventricular hemorrhage in premature infant. E. Usually take more than week to return to normal level following clinical resolution of bacterial infection in the newborn Ans ;AB A 6-year-old boy with a sore throat, cough, and fever is noted to have a marked elevation in his serum C-reactive protein. This finding indicates which of the following? A. Developing autoimmune reaction. B. Ineffective immune response C. Meningitis D. Non-specific inflammation E. Respiratory compromise Ans:-D C-reactive protein is one of the most commonly measured acute-phase reactants, which are a group of serum proteins showing a rapid increase in serum concentration in response to any inflammatory process. This finding is entirely non-specific & ndash; it only indicates a recent inflammatory process. An autoimmune reaction (choice A), which is certainly a concern with streptococcal pharyngitis, is suggested by the development of a rising ASO (antistreptolysin O) titer weeks after the illness. Acute phase reactants are not specific to autoimmune processes. Increases in C-reactive protein indicate a healthy immune response to an infective pathogen. An ineffective immune response (choice B) would not elicit acute-phase reactions. Meningitis (choice C) cannot be diagnosed from a blood test. The diagnosis of meningitis requires the appropriate clinical signs (neck stiffness, mental status changes) and a positive lumbar puncture.
  • 18. 2 ABDULRAHMAN BASHIR BENGHAZI CHILDREN HOSPITAL 18 25.1.2018 Respiratory compromise (choice E) produces changes in arterial blood gases and blood pH. Acute-phase reactants do not reflect respiratory status. Healthy term neonate differ from an adult in the following ways ; A. Less complement. B. Decreased IgG level. C. Fewer B lymphocytes. D. Lower level of secretory IgA. E. Higher level of C reactive protein Ans;AD The following are opsonine; A. IgD B. Complement C. C reactive protein D. Interferone-ά E. IgG Ans; BCE The following may indicate an underlying specific immunoglobulin deficiency: A. Recurrent oral thrush. B. Recurrent tonsillitis. C. Recurrent viral gastroenteritis. D. Recurrent conjunctivitis. E. Severe aphthous ulceration. Ans:-BC Oral thrush suggests cell-mediated defect. IgA deficiency or IgG sub-class deficiency may result in recurrent tonsillitis and viral gastroenteritis. In addition to abscesses and lymphadenopathy, mucus membrane infections such as conjunctivitis, rhinitis and stomatitis have been reported in chronic granulomatous disease. Oral ulceration may be: Common: aphthos, traumatic, hand, foot and mouth disease, herpangina, chemical burns (alkali, acid, aspirin). Uncommon: neutrophil defects (agranulocytosis, leukaemia, cyclical neutropenia), SLE, Behcet Syndrome, necrotising ulcerative gingivostomatitis, syphilis, Crohn's Disease, histoplasmosis. Conjunctivitis is common, since the conjunctivae react to a wide range of bacterial and viral agents, allergens, irritants, toxins, and they are inflamed in systemic diseases. Which of the following is true concerning immunity to viruses? A. IgA can offer protection at mucosal surfaces. B. Cytotoxic T cells are activated before natural killer cells during the course of infection. C. Viruses stimulate the non-immune cells that they infect to produce interferon-γ. D. Non-enveloped viruses are susceptible to damage by complement. E. Influenza virus can avoid antibody recognition by mutational changes in its nucleocapsid proteins. Ans:-A Natural killer cells are activated faster than cytotoxic T cells. Infected non-immune cells produce interferon-α and – β , whereas interferon-γ is produced by T cells. Influenza virus mutates its surface neuraminidase and haemagglutinin to avoid antibody recognition. Enveloped viruses are susceptible to complement attack Which of the organs below is considered an immunologically privileged site :-
  • 19. 2 ABDULRAHMAN BASHIR BENGHAZI CHILDREN HOSPITAL 19 25.1.2018 A. skin B. thymus C. bone marrow D. testicle E. pancrease Ans:- D Some tissues fail to elicit an immune response and are termed immunologically privileged sites- brain, cornea, testis, uterus and arguably liver. Several factors may contribute to immune privilege. The blood-brain barrier may block entry of lymphocytes into this organ and the avascularity of the cornea may also limit antigen/lymphocyte interaction. The eye also has high concentrations of TGF-beta which is a cytokine with immunosuppressive properties. Both the eye and the testis constitutively express Fas ligand. This membrane protein can bind to Fas on infiltrating leukocytes, and can induce apoptotic cell death in the Fas expressing cell. Liver transplantation may induce antigen-specific tolerogenic effects. The precise mechanisms are not clear, but have been attributed to the establishment of micro-chimerism, the existence of tolerogenic donor immature dendritic cells or their precursors within the liver graft. In contrast, skin is highly immunogenic. Until modern HLA typing techniques developed, skin grafting was used as a clinical test before solid organ transplantation to determine the tissue type compatibility between the donor and recipient. A patient has secondary antibody deficiency due to B-cell suppression. His primary pathology is likely to be A. Waldenstrom macroglobulinemia. B. X-linked agammaglobulinemia. C. immunoglobulin A deficiency. D. immunoglobulin M deficiency. E. common variable immunodeficiency Ans: A Antibody immunodeficiency can be primary or secondary. Secondary antibody deficiency due to B-cell suppression can occur with multiple myeloma, Waldenstrom macroglobulinemia, or chronic lymphocytic leukemia. Secondary deficiencies leave patients susceptible to the same pathogens that can cause recurrent pneumonia in patients with primary antibody deficiencies Primary antibody deficiencies include X-linked agammaglobulinemia, common variable immune- deficiency, selective immunoglobulin A or immunoglobulin M deficiency, and hyperimmunoglobulin M immunodeficiency. These disorders are characterized by chronic or recurrent pyogenic infection, especially pneumonia, caused by encapsulated bacteria (eg, S pneumoniae, H influenzae, and S aureus) and P aeruginosa . Untreated or recurrent pneumonia may lead to bronchiectasis. Regarding the early immunological development of the infant; A. In the newborn , there is significant quantity of circulating IgM B. IgG is actively transported across the placenta C. In the normal newborn , neutrophils function is reduced D. Large thymus gland detected on chest X ray is normal finding E. Circulating immunoglobuline level will have reached normal adult level by 6 month Ans ; BCD Which of the following statements is true concerning immunity to bacteria? A. Antibodies to secreted bacterial products play no protective role. B. Bacteria opsonised by antibodies and complement are more effectively phagocytosed than those opsonised by antibodies alone .
  • 20. 2 ABDULRAHMAN BASHIR BENGHAZI CHILDREN HOSPITAL 20 25.1.2018 C. Humoral rather than cellular immunity is predominant in protection against all types of bacteria. D. Phagocytes cannot engulf bacteria in the absence of antibodies . E. Endotoxin induces shock mainly through the activation of T cells . Ans:-B Antibodies such as those to cholera, diphtheria and tetanus toxin can play a major role in protective immunity in these infections. Cellular immunity is essential in protection against intracellular bacteria, eg mycobacteria. Phagocytes interact directly but weakly with bacteria, or strongly if they are complement- opsonised. Endotoxin activates macrophages by binding to CD14. The following the need for investigation of immune function ; A. Several episodes of URTI and GE in the space of one year B. Two episodes of bacterial meningitis C. Absence of tonsil in the second year of life D. History of chronic diarrhea and oral candidiasis E. History of first cousin marriage in the parents of child with recurrent pneumonia Ans; BCDE The following infection are appropriately paired with relevant immune deficiency; A. Recurrent pneumococcal infection and complement deficiency B. Recurrent viral infection and hypogammaglobulinemia C. Cutaneous abscess and Ig A deficiency D. Osteomyelitis and CGD E. Disseminated viral infection and T – cell deficiency Ans;ADE the following statement concerning immunity are correct ; A. decreased cellular immunity may lead to increased susceptibility to fungal Infection. B. hypogammaglobulinemia may lead to increased susceptibility to infection gram positive cocci C. IgG level are usually high from the age of 3 -12 month D. decreased cellular immunity has recognized association with hypoparathyroidism E. the immunologic defect in chronic granulomatous disease is reduced efficacy of gammglobuline Ans ;ABD Which of the following definitely excludes antibody deficiency? A. Normal serum immunoglobulins. B. Good IgG antibody responses to immunisations. C. The presence of existing antibody responses to past infections. D. Normal IgG subclasses. E. Normal peripheral blood lymphocyte subpopulations Ans:-B Normal immunoglobulins, including subclasses, do not exclude antibody deficiency. Hence in patients with good history of recurrent (proven) bacterial infections, responses to Haemophilus influenzae, Pneumococcus spp. and tetanus toxoid should all be assessed, as should postimmunisation responses if required. Antibodies to past infections and haemagglutinins can be helpful in assessing a patient. Of the following options, the best initial test for a workup for immunodeficiency is: A. Serum levels of IgG, IgM, IgA, IgE. B. A complete blood count with differential. C. Peripheral T-cell phenotyping. D. Erythrocyte sedimentation rate. E. Postimmunization immunoglobulin levels Ans:-B The initial step in the workup of a child with a suspected immunodeficiency should always include a complete blood count and differential, as this test will reveal disorders involving lymphopenia (particularly
  • 21. 2 ABDULRAHMAN BASHIR BENGHAZI CHILDREN HOSPITAL 21 25.1.2018 T-cell lymphopenias) and neutropenia. Other hints can be found by the CBC, including small platelets associated with the Wiskott–Aldrich syndrome, anemias, and evidence of eosinophilia. An erythrocyte sedimentation rate is a nonspecific marker of inflammation and would not point to a specific immunodeficiency. While quantitative immunoglobulins, peripheral T-cell phenotyping, and lack of antibody response to standard immunizations would all be part of an immunodeficiency investigation, these would not be considered first steps in the evaluation. The following are general feature of primary immunodeficiencies ( PID ); A. The overall incidence is very similar to that of phenylketonuria B. Failure to thrive is commonly associated feature. C. Bone marrow transplantation is indicated in all T & B lymphocyte function deficiency. D. PID should be suspected in any child who has had two ( or more ) invasive bacterial infections. E. PID commonly present with infection of increased severity Ans:-ABDE The following statement(s) concerning inherited immunodeficiency syndromes are true: A. The commonest inherited form of severe combined immune deficiency is x-linked B. The incidence of lymphoid malignancy is increased in ataxia telangiectasia C. The defect in Chediak-Higashi syndrome is in T-cell activation D. Deficiency in glucose-6-phosphate dehydrogenase results in impaired motility of leukocytes E. The molecular defect in X-linked hyper IgM syndrome is in the expression of CD40 on B-cells Ans:-AB Several defects can lead to the phenotype of severe combined immunodeficiency, the commonest inherited form is X-linked. There is a mutation in the gene encoding the common gamma chain – a chain shared by receptors for a number of interleukin cytokines. There is thus impaired T-cell differentiation as the interleukins are important T-cell growth and differentiation factors. Humoral immunodeficiency occurs because of a lack of T-cell help. The commonest autosomally inherited forms are adenosine deaminase and purine nucleotide phosphorylase deficiency. ADA and PNP are enzymes which catalyze the metabolism of purines via the uric acid pathway. Deficiency of either of these enzymes leads to the accumulation of toxic metabolites which severely affect lymphocyte production and function. Other known genetic defects that lead to severe immunodeficiency include mutations of DNA-dependent protein kinase (DNA-PK) and recombination activating genes (RAG). Lack of expression of MHC class II or MHC class I molecules also cause severe immunodeficiency. Ataxia telangiectasia is due to defects in ATM, a protein that is important in DNA repair. Patients have impaired cell mediated immunity, low IgE, IgA and IgG2 and are prone to respiratory infections and skin infections. Lymphoid malignancies as well as carcinomas occur with with increased frequency in these patients. Chediak-Higashi syndrome is an autosomal recessive disorder, characterised by giant granules in phagocytes due to a genetic defect in intracellular vesicle formation and hence impaired intracellular killing following phagocytosis. Other clinical features include partial oculocutaneous albinism and abnormal platelet function. Cytolytic T-cell- mediated killing is normal, but NK killing is impaired. G6PDH is an important enzyme in the hexose monophosphate shunt. This pathway maintains glutathione in a reduced state protecting the red cell membrane from oxidant stress . The enzyme also functions in the respiratory burst - a process important for intracellular killing by phagocytes. Complete G6PDH deficiency results in susceptibility to chronic bacterial and fungal infection - the clinical picture resembles a mild form of chronic granulomatous disease. X-linked hyper IgM syndrome is due to the lack of CD40 ligand (CD40L) expression on activated T- cells. This leads to impairment in immunoglobulin isotype switching and germinal centre formation as CD40/CD40L interaction is needed for these processes. Serum IgM levels are high with deficiency of IgG and IgA. Clinically, these patients are particularly susceptible to opportunistic lung pathogens such as Pneumocystis carinii. In primary immune deficiency:-
  • 22. 2 ABDULRAHMAN BASHIR BENGHAZI CHILDREN HOSPITAL 22 25.1.2018 A. In chronic granulomatous disease the infection is usually due to catalase producing bacteria. B. Severe combined immunodeficiency is an autosomal dominant disorder. C. In DiGeorge syndrome hypoadrenalism is part of the disorder. D. Bone marrow transplant is. the treatment of choice for Chediak-Higashi syndrome. E. Thrombocytosis is part of Wiskott Aldrich syndrome. Ans:-AD The following is true regarding therapy for primary immunodeficiencies ( PIDs ) ; A. gene therapy is indicated for common variable immunodeficiencies B. gamma – interferon can be useful in chronic granulomatous disease C. C1 esterase inhibitor concentrate for hereditary angioedema D. Bone marrow transplantation is indicated for severe combined immunodeficiency E. Immunoglobuline replacement is indicated for hyper IgM syndrome Ans ; BCDE Definitive prenatal diagnosis is possible for the following the primary immunodeficiencie A. Selective Ig A deficiency. B. Wiskott Aldrich syndrome C. X linked form of severe combined immunodeficiency. D. DiGeorge syndrome E. Common variable immunodeficiency Ans ; BCD During the last trimester, IgG is actively transported across the placenta to suppy passive immunity to the fetus. Which disease occurring during pregnancy is most likely to lead to the neonate having low immunoglobuline levels and hence being prone to bacterial infections. A. Intestinal lymphangectasia. B. Systemic lupus erythematosus. C. Myasthenia gravis. D. Ulcerative colitis. E. Prematurity. Ans:-A The following statements on cellular immunity are correct A. cytotoxic T cells (CTL) carry +8 glycoprotein and respond to peptides presented by HLA class 1 B. CTL produce interleukin 2 (IL-2). C. Di George syndrome is associated with impaired cellular immunity. D. Lymphopenia is a feature of ataxia telangiectasia. E. Wiscott—Aldrich disease is an autosomal recessive condition characterised by depressed cellular immunity Ans:- ACD T helper cells (Th) express 4+ and recognise HLA (major histocompatibility complex) class 2. Most CTL are 8+ and restricted to class 1 HLA. A sub-class of Th cells produce IL-2. Di George syndrome (thymic aplasia) is associated with impaired T cell production. Immunoglobulin levels are usually adequate and there is an increased risk of autoimmune disease. Ataxia telangiectasia has several deficiencies of immunoglobulins and lymphopenia: it is autosomal recessive. Wiscott—Aldrich syndrome is X-linked, the gene defect leading to abnormalities in the regulation of sialophorin (0043). The condition is characterised by thrombocytopenia and lymphopenia. A 1-year-old boy has a history of multiple episodes of otitis media, sinusitis, and pneumonia. He has not had any fungal, protozoan, or mycobacterial infections. You suspect a disorder of humoral immunity. Which of the following is the best initial screening test? A. Dihydrorhodamine (DHR) flow cytometry test. B. Quantitative serum immunoglobulins. C. Examination of the peripheral blood smear.
  • 23. 2 ABDULRAHMAN BASHIR BENGHAZI CHILDREN HOSPITAL 23 25.1.2018 D. Total hemolytic complement (CH50). E. Anergy panel Ans:-B Most primary immunodeficiency diseases involve abnormalities of immunoglobulin concentration or function (humoral immunity). The hallmark of immunoglobulin deficiency is increased susceptibility to sinopulmonary infections caused by encapsulated bacteria. Serum immunoglobulin concentrations should be measured in such patients. The dihydrorhodamine (DHR) flow cytometry test is used to diagnose chronic granulomatous disease, a disorder of phagocyte oxidative metabolism. Examination of the peripheral smear can yield valuable information about neutropenia, lymphopenia, or thrombocytopenia, which is useful and adjunctive but does not provide a definitive diagnosis. Total hemolytic complement assays are useful to exclude isolated deficiencies of complement components. Sinopulmonary infections are generally not associated with complement deficiency. Anergy panels are used to evaluate cell-mediated immune function. Which of the following pathogens are associated with the accompanying immune disorder? A. Mycobacteria and type 1 cytokine defects. B. Gram-negative bacteria and complement deficiency. C. Enterovirus and antibody defects D. Staphylococcus and complement deficiency. E. Meningococcus and neutrophil defect. Ans:-AC The pathogens associated with an underlying immune disorder are: • mycobacteria- type 1 cytokine defects • Gram-negative bacteria- neutrophil defect. • enterovirus- antibody defects as well as defects in cell –mediated immunity • Staphylococcus - neutrophil defect • Meningococcus - complement deficiency • Pneumococcus and Haemophiius influenzae B – antibody and complement defects • Salmonella- type 1 cytokine defect-, and Cell mediated defects. • Mycoplasma- antibody defects • herpes viruses- defects in cell-rnediated immunity. -------------------------------------------------------------------------------------------------------------------------------- The following is true of X – linked agammaglobulinemia ( Brutons disease ) ; A. Plasma cells are absent from bone marrow. B. it seldom presents before the age of 5 years. C. it commonly present with infection of the lungs and sinuses . D. the underling defect is an abnormality of the gene for B cell specific tyrosine kinase ( BTK ) E. early replacement immunoglobuline therapy can help to prevent chronic lung damage Ans ;ACDE A 3-year-old boy presented with recurrent attacks of pneumonia and otitis media since the 1st birthday. Your diagnosis is X-linked agammaglobulinemia. of the following, the MOST likely offending organismis A. CMV. B. Mycoplasma. C. Pneumocystis jiroveci. D. Staphylococcus aureus. E. Streptococcus pneumonia. Ans:-E
  • 24. 2 ABDULRAHMAN BASHIR BENGHAZI CHILDREN HOSPITAL 24 25.1.2018 A 4.5-year-old boy presented with history of recurrent attack of secretary otitis media and purulent nasal discharge, Haemophilus influenza revealed by culture of ear discharge, on physical examination there is no tonsillar tissue and no palpable lymph nodes. Of the following, the MOST appropriate test to confirm the diagnosis is measurement of A. flow cytometry. B. lgA concenteration. C. isohemagglutinins titer. D. IgG and lgM concenteration. E. antibodies to antigens of routine immunizations Ans:-A The diagnosis of X-linked agammaglobulinemia (XLA) should be suspected if lymphoid hypoplasia is found on physical examination (minimal or no tonsillar tissue and no palpable lymph nodes). Flow cytometry is an important test to demonstrate the absence of circulating B cells, which will distinguish this disorder from common variable immunodeficiency, the hyper-lgM syndrome, and transient hypogammaglobuiinemia of infancy. In X-linked agammaglobulinaemia all of the following are true except: A. B cells are absent. B. It is X-linked recessive and is caused by defects in a gene located at Xq22. C. The most common organisms causing infection are Staphylococcus aureus and N. meningitidis. D. Infection with Giardia lamblia is not uncommon. E. Neutropenia may be present, particularly during severe infections. Ans:-C This affects males, with a repeated history of bacterial infection. It usually occurs after 6 months of age and before 2 years of age. Mild forms may present later in childhood, or even in adolescence or early adulthood. The most frequent sites of infection are otitis, sinusitis and pneumonia, and the most common organisms are Haemophilus influenzae and Streptococcus pneumoniae. Non-infectious manifestations (e.g. arthritis) are unusual in XLA, but have been described. Pneumocystis carinii pneumonia is very rare, but infection with Giardia lamblia is not uncommon. Neutropenia may be present during severe infections, but not other times. All isotypes of immunoglobulins are absent or very low. Circulating mature B cells are absent, as are specific antibodies. T cells are normal. Children with XLA will usually have complications, e.g. bronchiectasis, deafness, enteroviral meningo-encephalitis and neutropenia. The treatment is lifelong immunoglobulin replacement every 6–8 weeks and maintenance of IgG to the level of IgG at 8 g/l. Infections should be treated, and physiotherapy should be given for bronchiectasis. Referral should be made to a geneticist for all boys with XLA, as there will be risk for their offspring. All of their daughters will be carriers, but sons will be normal. A couple brings their son in to a specialty clinic for evaluation of recurrent bacterial infections involving the respiratory tract. Other family members have a similar disorder, as noted in the pedigree above. Which of the following is the most likely diagnosis? A. Bruton's agammaglobulinemia. B. Common variable immunodeficiency. C. DiGeorge syndrome. D. Hereditary angioedema. E. Isolated IgA deficiency Ans:-A Several immunodeficiency disorders have X-linked genetics, including Bruton's agammaglobulinemia, Wiskott-Aldrich syndrome, and some cases of severe combined immunodeficiency diseases. Bruton's agammaglobulinemia is characterized by recurrent respiratory infections caused by pyogenic organisms. Common variable immunodeficiency (choice B) is a relatively common, but probably heterogeneous, group of acquired and familial diseases. It is distinct from X-linked (Bruton's) agammaglobulinemia.
  • 25. 2 ABDULRAHMAN BASHIR BENGHAZI CHILDREN HOSPITAL 25 25.1.2018 DiGeorge syndrome (choice C) is due to a developmental malformation of the third and fourth pharyngeal pouches leading to failure of the thymus, and sometimes the parathyroids, to develop. Defective cellular immunity and abnormalities of calcium metabolism are typical. Hereditary angioedema (choice D) is a usually recessive genetic disease caused by deficiency of C1 esterase inhibitor. Isolated IgA deficiency (choice E) can be acquired or genetic, but is not usually X-linked. An 18 month old male presents with recurrent pneumonia ear infections and tonsillitis.. The family history includes a previous male infant death at 2 years of age from pnumococcal meningitis. The parents are therefor verv concerned and wish further investigations to be carried out. After the result have returned, it is found that the child has absent or decreased level of immunoglobulins A, G and M. He has reduced response to blood group antibodies to immunisations and an increased percentage of E rosette with red blood cells. His PHA and nitrobiue tetrazolium (NBT) test are norma1 . The ultimate disorder is X-linked agammag!obulinemia. Which of the following are appropriate treatments (more than one answer may be given)? A. A course of benzylpenicillin for an acute pneumonia and tonsillitis. B. Azithromycin for 3 days every fortnight as aprophylctic antibiotic C. Intravenous immunoglobulin. D. A booster vaccine to DPT, Hib and meningitis C. E. Bone marrow transplantation. Ans:-BCDE Prenatel diagnosis is possible. Investigations show a profound failure of B cell d evelopment so plasma cells from the bone marrow are absent. There is an abnormal B-cell function and number so there are isohaemagglutinins or surface immunoglobulins. Investigations undertaken include those stated. Including finding an increase in OKT3 (49-63% of cases). Neutrophil, lymphocyt, PHA., nitroblue tetrazolium (NBT), sweat test and T-cell function are all normal. The PHA test measures T-cell proliferation. The T- cell lymphocyte s are identified by their ability to form rosettes with sheep red blood cells.T-ceII proliferation gives the total percentage of lymphocytes; the number is increased if no B-cells are present. Monoclonal antibodies are now used to identify and characterize T-cells. Monoclonal antibodies against T3 receptors measure the total Tcell number, T4 receptors, the T helper Cell population and T8 receptors in the T suppressor/cytotoxic (Ts/c) group. Treatment includes prophylactic antibiotics and intravenous immunoglobulins. Bone marrow transplantation is carried out if medical intervention fail. A 7-month-old male presents to your office with chronic enteroviral infection and failure to thrive. The mother is very concerned as multiple family members have died of infection in infancy in the past. A complete blood count demonstrates a normal total white blood cell count, but further investigation with flow cytometry detects normal numbers of T cells and the absence of any circulating B cells. The patient’s diagnosis is most likely to be which of the following? A. Severe combined immunodeficiency. B. 22q11 syndrome. C. X-linked agammaglobulinemia. D. Adenosine deaminase deficiency. E. Vertically transmitted HIV infection Ans:-C Normal T-cell numbers but absent B cells indicate a primary disorder of the humoral immune system leading to B-cell maturation arrest. The most common of these is X-linked agammaglobulinemia, which accounts for over 80% of these disorders. The significant family history of early death from infection is a clue that this patient’s disorder may be genetic in origin. XLA is caused by an abnormality in Bruton’s tyrosine kinase resulting in a block preventing maturation past the CD19+ pro-B cell stage. The lack of B cells in the periphery leads to susceptibility with typical organisms dependent on humoral immune system
  • 26. 2 ABDULRAHMAN BASHIR BENGHAZI CHILDREN HOSPITAL 26 25.1.2018 function. The other listed disorders all result in T-cell abnormalities, which ultimately cause abnormalities in B-cell function, but cause disorders in both cellular and humoral immune system function. Which of the following definitely excludes antibody deficiency? A. Normal serum immunoglobulins. B. Good IgG antibody responses to immunisations. C. The presence of existing antibody responses to past infections. D. Normal IgG subclasses. E. Normal peripheral blood lymphocyte subpopulations Ans:-B Normal immunoglobulins, including subclasses, do not exclude antibody deficiency. Hence in patients with a good history of recurrent (proven) bacterial infections, responses to Haemophilus influenzae, Pneumococcus spp. and tetanus toxoid should all be assessed, as should postimmunisation responses if required. Antibodies to past infections and haemagglutinins can be helpful in assessing a patient Which of the following isn't highly suspicious of immunodeficiency A. Chronic sinopulmonary infection B. Continual infection without period of good health C. Unusual infecting agent D. Incomplete response to treatment E. Eczema Ans ;E A young mother brings her 2 year old son to the doctor and comments that he is always sick. Upon further questioning, his mother is adament he was healthy until he was about 9 months old when the problems began. He has had recurrent respiratory infections due to pneumoccocus and Haemophilus influenzae. He also has recurrent bouts of ear infections and pharyngitis. His serum concentrations of immunoglobulins are in the third percentile. He is eventually diagnosed with X-linked agammaglobulinemia of Bruton. This is a problem with B lymphocytes. Which of the following is a true feature of B lymphocytes? A. their immunocompetence is gained in the thymus B. they are found in the deep cortical region of lymph nodes C. presence of surface immunoglobuin D. forms rosette with sheep red blood cells E. they make up 70% of the lymphocytes in the circulation Ans:- C Surface immunoglobulins are seen with B lymphocytes. All of the other characteristics listed (choice a, b, d, and e) are characteristics of T lymphocytes.Choice (a) is incorrect because B lymphocytes do not become immunocompetent in the thymus; they gain immunocontpetence in the bursa equivalent (postulated to be the bone marrow).B lymphocytes are located in the germinal centers and the medullary cords of a lymph node, therefore choice (b) is incorrect. B lymphocytes do no form rosettes with sheep red blood cells, T lymphocytes do. B lymphocytes comprise 30% of the circulating lymphocytes, therefore choice (e) is incorrect. Patients with disorders of B-cell development are susceptible to infections from encapsulated organisms and enteroviral infections. These patients typically become symptomatic at what age? A. In the second decade of life. B. Between the ages of 5 and 10. C. In the first three months of life. D. Between the first 6 and 12 months of life. E. In the immediate postpartum period. Ans:-D
  • 27. 2 ABDULRAHMAN BASHIR BENGHAZI CHILDREN HOSPITAL 27 25.1.2018 Disorders of B-cell development result in maturation arrest of B cells and lack of mature B cells in the peripheral circulation. This condition results in agammaglobulinemia. The affected child depends on maternal immunoglobulin for humoral immune system protection; however once maternal immunoglobulin decays (typically by 6 months of age), the child has no protection against infections with encapsulated bacteria or viruses, such as enterovirus, leading to chronic recurrent infection Hypogammglobulinemia is manifested by all of the following except : A. Recurrent otitis media, pneumonia and meningitis are the most common infection B. Pneumococcus and Haemophilus influenza are the most common infecting agent C. Patient are often clinically well between episodes of infection. D. Infant with X-linked agammaglobulinemia are at increased risk of infection with vaccine induced polio and echovirus encephalitis. E. Hypogammaglobulinemia is present when the total serum immunoglobuline are < 250 mg/dl Ans ; C A child has a history of recurrent infections with organisms having polysaccharide antigens (i.e., Streptococcus pneumoniae and Hemophilus influenzae). This susceptibility can be explained by a deficiency of:- A. C3 nephritic factor B. C5 C. IgG subclass 2 D. myeloperoxidase in phagocytic cells E. secretory IgA. Ans:-C IgG is the predominant antibody in the secondary immune response. IgG subclass 2 is directed against polysaccharide antigens and is involved in the host defense against encapsulated bacteria. C3 nephritic factor (choice A) is an IgG autoantibody that binds to C3 convertase, making it resistant to inactivation. This leads to persistently low serum complement levels and is associated with Type II membranoproliferative glomerulonephritis. C5 (choice B) is a component of the complement system. C5a is an anaphylatoxin that effects vasodilatation in acute inflammation. It is also chemotactic for neutrophils and monocytes and increases the expression of adhesion molecules. A deficiency of C5a would affect the acute inflammatory response against any microorganism or foreign substance. Myeloperoxidase in phagocytic cells (choice D) is an element of the oxygen-dependent pathway present in phagocytic cells that effectively kills bacterial cells. The hydrogen peroxide-halide complex is considered the most efficient bactericidal system in neutrophils. Chronic granulomatous disease is associated with a deficiency of NADPH oxidase, which converts molecular oxygen to superoxide (the first step in the myeloperoxidase system). Patients are susceptible to granulomatous infections and staphylococcal infections. Secretory IgA (choice E) is the immunoglobulin associated with mucous membranes. Selective IgA deficiency is the most common hereditary immunodeficiency. In this disorder, there is failure of the B cell to switch the heavy chain class from IgM to IgA. Patients have an increased incidence of sinopulmonary infections, diarrhea, allergies, and autoimmune diseases. A 7-month-old infant is brought to your office because his mother believes that he has been “sick for months.” The child appears to have bilateral otitis media as well as coarse rhonchi in his lungs, but he is active and appears to be gaining weight. A complete blood count reveals normal numbers of T
  • 28. 2 ABDULRAHMAN BASHIR BENGHAZI CHILDREN HOSPITAL 28 25.1.2018 and B cells, but quantitative immunoglobulins demonstrate profoundly low levels of IgG, IgM, and IgA. Your next step would be which of the following? A. Refer the patient for bone marrow transplant. B. Start scheduled intravenous immunoglobulin administration. C. Initiate prophylactic antibiotic therapy. D. Reassure the mother that the child should recover from immune function. E. Anticipate that the child will develop chronic lung disease Ans:-D This patient has hypogammaglobulinemia, leading to chronic infection now that the placentally-acquired immunoglobulin from his mother has decayed. As contrasted with X-linked agammaglobulinemia, however, this patient has normal circulating numbers of B cells and the low levels of immunoglobulin are due to a delay in onset of Ig production. This disorder is called transient hypogammaglobulinemia of infancy. As depicted by the title, this disorder is not permanent; spontaneous recovery is expected by 2 to 4 years of age. Prophylactic antibiotics are not indicated, nor is prophylactic intravenous immunoglobulin administration unless the patient continues to have serious recurrent infection. In hypogammaglobulinaemia, which of the followings are true? A. In nephrotic syndrome, the loss of immunoglobulin in urine is the only cause of predisposition to infection. B. In common variable immunodeficiency, there is an increased incidence of lymphoma. C. Selective IgA deficiency is the commonest inherited form. D. In patients over 40 years of age, a chest radiograph is mandatory to exclude Thymoma E. The genetic defect in X-linked agammaglobulinaemia (XLA) is in Bruton’s tyrosine kinase (Btk) Ans:- BCDE In nephrotic syndrome, the susceptibility to pneumococcal infection is multifactorial and includes decreased immunoglobulin levels, immunosuppressive treatment and in those patients with immune complex mediated glomerulonephritis -a major factor is decreased levels of complement. In XLA or Bruton’s agammaglobulinaemia, B-cell maturation is arrested at the pre-B-cell stage, suggesting that Btk may be required to couple the intracellular signaling events mediated by the pre-B cell receptor. Selective IgA deficiency is the commonest form of inherited Ig deficiency (Incidence about 1 in 800). Although usually asymptomatic, the incidence is higher in patients with chronic lung disease suggesting that IgA deficiency may be associated with increased respiratory infections. Selective IgA deficiency can also occur after phenytoin and barbiturate administration as well as post toxoplamosis, measles or other viral infections. Some patients with “selective” IgA deficiency may also have deficiencies in IgG2 and IgG4. The commonest form of immunoglobulin deficiency is however, transient hypogammaglobulinaemia of infancy. This presents between 6-12 months when the maternal antibody levels wane and the infant’s own humoral system is not yet fully developed. IgG subclass deficiency is associated with the following except: A. IgG2 is associated with low IgA. B. IgG2 is associated with poor antibody responses to polysaccharide antigens. C. It is usually a permanent problem. D. Severe IgG2 deficiency may lead to bronchiectasis. E. Prophylactic antibiotics can be used to prevent severe infections Ans:-C This is a very rare condition that sometimes may be responsible for primary severe immune deficiency. Children usually present with upper respiratory tract infection. It can be familial, and repeated infection may cause bronchiectasis. It usually resolves spontaneously, and immunoglobulin is not needed except in cases with severe recurrent infection. Prophylactic azithromycin or Augmentin is advisable. No routine blood tests or chest X-rays are needed.
  • 29. 2 ABDULRAHMAN BASHIR BENGHAZI CHILDREN HOSPITAL 29 25.1.2018 You are evaluating a 1-year-old male in your office in post-hospitalization follow-up. He was admitted with Pneumocystis jiroveci pneumonia 2 weeks ago. You also note that he has had several respiratory infections with H. influenza and pneumococcus despite being immunized. Laboratory evaluation notes a normal CBC. Flow cytometry demonstrates normal numbers of T and B lymphocytes, but quantitative immunoglobulins show significantly decreased IgG and IgA levels, although IgM levels are elevated. This immunodeficiency in most likely caused by which of the following? A. Maturation arrest of B cell precursors. B. Abnormal function of CD4+ T helper cells. C. Abnormal NK cell function. D. Elevated levels of IgE. E. Impaired class-switch recombination of immunoglobulins Ans:-E The elevated levels of IgM and profoundly low levels of IgG and IgA indicate a failure in immuneglobulin class switching, making the patient susceptible to encapsulated organisms. The most common of these disorders is X-linked hyper-IgM syndrome which causes an abnormality in the gene encoding the CD40 ligand on T cells leading to failure of CD40 on B cells leading to blockade of the class-switch process. CD40 is also present on macrophages and dendritic cells, leading to susceptibility to opportunistic organisms such as P. jiroveci. X-linked hyper IgM syndrome :- A. Causes defective B cell development B. Is associated with normal levels of IgG C. Can present with pneumocystis carinii infection D. Is due to a defective CD40 ligand E. Is due to a failure of Ig isotype switching Ans:- CDE The hyper-IgM syndrome is a rare, inherited immune deficiency disorder resulting from defects in CD40 ligand/CD40 interaction. X-linked hyper-IgM (type 1) is caused by defects in the CD40 ligand expressed on activated T-cells. Autosomal recessive hyper-IgM (type 2) is caused by defects in the CD40-activated RNA-editing enzyme- activation-induced cytidine deaminase. This enzyme is required for immunoglobulin isotype switching and somatic hypermutation in B-cells. In X-linked hyper-IgM, the loss of interaction between CD40 and its ligand results in impairment of T-cell, B-cell and macrophage function. The interaction of CD40L on T-cells and CD40 on B-cells is essential for B-cell activation and proliferation, isotype switching and formation of germinal centres (which are required for affinity maturation of immunoglobulin). Therefore, although there are high levels of serum IgM which can be produced without T-cell help, the T-dependent B-cell response is defective, and there is a lack of IgG, IgA or IgE production. CD40L/CD40 interaction is also necessary for optimal activation of macrophages by T-cells. The subsequent macrophage dysfunction contributes to the profound immunodeficiency seen in these patients. Patients are susceptible to infection with extracellular bacteria as well as intracellular pathogens. Boys present with recurrent sinopulmonary infections and chronic diarrhea often caused by Giardia, Campylobacter, rotavirus, or Cryptosporidium. These patients are particularly prone to opportunistic lung infections caused by Pneumocystis carinii. There is an increased incidence of autoimmune diseases and malignancies including adenocarcinomas of the liver and biliary tract. Laboratory studies usually reveal normal B- and T-cell numbers, however neutropenia is common. IgM levels are elevated but IgG and IgA levels are low. Diagnosis is confirmed by identifying the defective gene. Treatment is supportive with IVIG and antibiotics. A 14-year-old male with X-linked lymphoproliferative disease is admitted to the hospital with fulminant Epstein–Barr viral infection, hepatitis, and respiratory failure. He is currently intubated,
  • 30. 2 ABDULRAHMAN BASHIR BENGHAZI CHILDREN HOSPITAL 30 25.1.2018 on ventilator and pressor support in the Pediatric Intensive Care Unit. He has received 2 g/kg of intravenous immunoglobulin. Should he survive this current illness, you would expect which of the following? A. He is at great risk for extranodal lymphoid malignancy. B. He will have a reactive, sustained polyclonal hypergammopathy. C. He will develop chronic, active hepatitis. D. His female offspring will be at great risk for fulminant EBV infection. E. He will suffer significant neurologic impairment Ans:-A X-linked lymphoproliferastive disease is due to a mutation in SH2D1A, which encodes the protein SAP (signaling lymphocyte activation molecule-associated protein), which regulates T cell response to EBV infection. Patients can develop overwhelming EBV infection, which is fatal in over 50% of patients infected. Survivors have sustained hypo- or agammaglobulinemia and can often develop extranodal lymphoid malignancies, particularly of the Burkitt type. Other than as residua of severe infection, there are no specifically associated neurocognitive abnormalities and patients do not develop chronic hepatitis --------------------------------------------------------------------------------------------------------------------------------- Patients with IgA deficiency should be given which of the following with caution? A. Packed RBCs. B. Intravenous Immunoglobulin. C. GM-CSF. D. Pneumococcal immunization. E. Folic acid supplementation Ans:-B IgA deficiency is the most common primary immunodeficiency, occurring in approximately 1 in 600 people. While most people with selective IgA deficiency are asymptomatic, some develop recurrent infections at mucosal barriers, such as the respiratory and gastrointestinal tracts. Some patients with IgA deficiency develop anti-IgA antibodies and are at risk for anaphylaxis when they receive any blood product containing IgA, the most common of which is pooled intravenous immunoglobulin. A 6-year-old male with juvenile idiopathic arthritis is brought to your office by his mother because she is worried that his arthritis medicine “keeps making him sick.” He is currently taking naproxen, calcium, and vitamin D. On review of his medical records, you note that he has had multiple respiratory infections in the past several years, as well as a history of chronic otitis media requiring tympanostomy tube placement which pre-dates his arthritis diagnosis. You suspect that he might have an underlying immunodeficiency. Your advice to the mother would be which of the following? A. The arthritis medication is worsening his immunodeficiency and will need to be stopped. B. The arthritis and the immunodeficiency are unlikely to be related. C. More aggressive treatment of his arthritis may increase the patient’s risk of infection. D. Treatment of the arthritis will result in an improvement in his immunodeficiency. E. The patient will require IgA supplementation. Ans:-C Given the chronic recurrent respiratory tract infections in an otherwise healthy child with arthritis, this patient is likely to have IgA deficiency. The relationship of autoimmune disease and IgA deficiency has been well described, particularly with juvenile idiopathic arthritis and celiac disease. There is no treatment for IgA deficiency and treatment of the arthritis will not improve IgA production. Given the destructive nature of untreated JIA, the medication cannot be stopped. More aggressive treatment for arthritis, such as with methotrexate, steroids, or biologic agents will lead to further immunosuppression and the patient will need to be monitored for infection and treated aggressively when present. IgA deficiency:-
  • 31. 2 ABDULRAHMAN BASHIR BENGHAZI CHILDREN HOSPITAL 31 25.1.2018 A. Is an extremely rare finding. B. May resolve spontaneously. C. Can lead to increased respiratory & GIT infection D. Is associated with autoimmune conditions. E. Is commonly found in association with IgG3 subclass deficiency. Ans:- BCD In IgA deficiency, patients with more severe and recurrent sinopulmonary infection tend to have an associated IgG2/IgG4 or IgG4 subclass deficiency. Selective IgA deficiency:- A. Is treated with iv sandoglobulin. B. is very rare (incidence less than 1 in 10,000). C. May present with autoimmune disease. D. IgG2 subclass deficiency may also be present with more severe infective complications. E. is associated with HLA-DR3. Ans:-CDE Selective IgA Deficiency is the commonest immune deficiency (affects 1 in 500-700 people). It is usually sporadic, but can be inherited in an autosomal dominant or autosomal recessive pattern. There is an association with MHC alleles HLA B8, DR3 and deletions of the C4A gene. Patients are often asymptomatic. If present, clinical features are usually mild – recurrent sinusitis and respiratory infections (mild) or chronic diarrhoea. Autoimmune syndromes (e.g. rheumatoid arthritis, Sjogrens disease, pernicious anaemia, thyroiditis) are associated. Autoantibodies (organ-specific and non-specific) are often found even in the absence of clinical disease. Anti-IgA antibodies occur in 30-40% and these may cause anaphylaxis if transfusions are administered. Serum IgA levels are usually < 50µg/ml but serum IgG and IgM levels usually normal. Some patients may have additional IgG subclass deficiencies. The defect in this syndrome is a block in differentiation of IgA-expressing B cells to antibody- secreting plasma cells. Treatment is symptomatic – antibiotics for infections. IgA deficiency;- A. Typically lead to sever Sinopulmonary infection. B. Is associated with autoimmune disease. C. Should be treated with intravenous immunoglobulin therapy. D. Is feature of hereditary angio-oedema. E. Is associated with atopy. Ans; BE Isolated IgA deficiency is the most common primary immunodeficiency. Most individual remain asymptomatic, and those with recurrent infection tend to improve with age. Minor recurrent sinopulmonary infection are the most common complication in childhood, however sever pneumonia and trouble ear infection can occur. Infections at other sites are relatively rare and include increase the incidence of Giardiasis. Allergic disease occur with an increased frequency in patient with immune deficiency IgA deficiency:- A. Is seen in Kartagener's syndrome B. Is usually asymptomatic C. May delay diagnosis of coeliac disease D. Spontaneous recovery is a recognised feature E. Affects both serum and secretory IgA levels Ans:- ABCDE IgA deficiency may be seen in: i) CVID (all Ig groups) ii) Bruton's agammaglobulinemia (all Ig groups)
  • 32. 2 ABDULRAHMAN BASHIR BENGHAZI CHILDREN HOSPITAL 32 25.1.2018 iii)Transient hypogammaglobulinemia of infancy (IgG,M+A) iv) Ataxia telangiectasia (IgA+E) v) Bloom's syndrome (IgA+M) vi) Kartagener's syndrome (IgA) vii) Selective IgA deficiency (IgA) Selective IgA deficiency is common (1:350) but the incidence is not universal worldwide (rarer in Japan 1:1,800), affecting the levels of IgA (A1, A2 & secretory forms). Most people are completely asymptomatic, but occasionally there is an increased incidence of respiratory, GI and GU infections. Increasingly the diagnosis is been made following investigation of FTT where the diagnosis of coeliac disease is made based on serology (antigliadin & endomyesial IgA) and therefore it is worth bearing in mind in a negative coeliac screen where malabsorption seems likely and other conditions are similarly ruled out. Alternatively, pre- and post- jejunal biopsy should be performed. In selective IgA deficiency there is an increased incidence of cows' milk allergy, anaphylaxis, autoimmune disease and malignancy. In transient hypogammaglobulinemia of infancy following the erosion of maternal passive immunity there may be a delay of up to 36 months before full humoral immunity is restored. The levels of IgG & A are predominantly affected, but also IgM. Selective IgA deficiency associated with:- A. Autosomal dominant B. Associated with rheumatoid arthritis C. Lymphoma is recognized complication D. Autosomal recessive Ans:-ABC Selective IgA deficiency associated with:- A. Ataxia telangectasia B. Chronic mucocutaneous candidiasis C. Rheumatoid arthritis D. Phenytoin Ans:- ACD Selective IgA deficiency associated with:- A. INH B. Penicillamine C. Tetracycline D. Phenytoin Ans:-BD IgA deficiency is noted in patients treated with the same drugs associated with producing CVID (phenytoin, d-penicillamine, gold, and sulfasalazine), suggesting that environmental factors may trigger this disease in a genetically susceptible person. IgA deficiency; A. Is the commonest form of primary immunodeficiency B. Usually present as growth faltering C. Has an association with celiac disease D. Is contraindication to BCG immunization E. Increase the risk of transfusion reaction Ans;-ACE Selective IgA deficiency A. Is inherited as an X- linked dominant B. Occurs in about 1 in 400 000 people C. Severs T cell deficiency almost always coexist
  • 33. 2 ABDULRAHMAN BASHIR BENGHAZI CHILDREN HOSPITAL 33 25.1.2018 D. Routine gammglobuline is the treatment of choice E. Patient have an increased incidence of allergy, sinopulmonary infection gastrointestinal disease and autoimmune disease Ans:-E IgA deficiency: A. is the commonest form of primary immunodeficiency. B. usually presents as growth faltering. C. has an association with coeliac disease. D. Has contraindication to BCG (bacille CalmetteGuerin) immunization. E. increases the risk of transfusion reactions. Ans:-ACE A 17-month old girl presented with history of recurrent attacks of pneumonia since the age of 8 months. You suspect a predominant B-cell defect. Of the following, the BEST simple initial screening test is A. lgA measurement. B. lgG measurement. C. lgM measurement. D. lgE measurement. E. isohemoagglutinins titre Ans:-A If the lgA level is normal, selective lgA deficiency, which is the most common B cell defect, is excluded, as are most of the permanent types of hypogammaglobulinemia, as lgA is usually very low or absent in those conditions. if lgA is low, lgG and lgM should also be measured. A 3 year-old girl presented with recurrent attacks of chest infection and frequent episodes of chronic diarrhea, stool examination revealed Giardia lamblia and serum IgA level was very low. Regarding this condition, all the following are trueEXCEPT A. autosomal dominant inheritance. B. phenotypically normal blood B cells. C. administration of IVIG is not indicated. D. environmental factors may trigger the disease. E. it is associated with a celiac-like syndrome with dramatic response to gluten free diet Ans:-E An isolated absence or near absence (<10 mg/dl.) of serum and secretory lgA is the most common well- defined immunodeficiency disorder. lgA deficiency is associated with a celiac-like syndrome, which may or may not respond to a gluten-free diet. --------------------------------------------------------------------------------------------------------------------------------- Which of the following is associated with common variable immunodeficiency? A. Inflammatory bowel disease B. Autoimmune disease C. Normal levels of IgA D. Granulomatous lesions E. Susceptibility to recurrent parasitic infections Ans:- ABD CVID refers to a spectrum of disorders in which cell-mediated and antibody deficiency are combined in variable amounts. Immunological findings are variable, but it is the humoral immune deficits that usually present clinically. Some T-cell functional assays may be abnormal. Patients present with recurrent pyogenic infections, and in some cases susceptibility to infections with Pneumocystis carinii, viruses or Candida. Autoimmune disease occurs in about 10% of cases, usually immune thrombocytopenia, haemolytic anaemia or SLE. There is an increased incidence of malignancy. Granulomas are a special feature of CVID